A Beautiful Mind 2010 Medical Ethics Conference

You might also like

Download as docx, pdf, or txt
Download as docx, pdf, or txt
You are on page 1of 53

Wiznitzer 1

A Beautiful Mind- Jewish Approaches to Mental Health


The Fifth Annual Medical Ethics Conference
Sunday, October 31, 2010

Welcome Address by Adiel Munk and Jennifer Kraut

Honored speakers and esteemed guests, welcome to the Fuld Family Conference, “A Beautiful
Mind: Jewish Approaches to Mental Health”- as copresidents and conference organizers, we’d
like to take this opportunity to thank you for your support in this year’s conference. Relates to
mental health through the prism of Jewish law and Torah values. Want to thank the Fuld Family-
privileged to have Rabbi Dovid and Mrs. Anita Fuld here today. It is with their support that we
were able to create today’s conference with the highest sensitivity and professionalism in mind.

Sefer Breishit with the emotional experiences and relationships of our Avot and Imahot. Chief
Rabbi Lord Sir Jonathan Sacks compare way Avraham and Noach respond to tragedy and grief.
Noach began to be a man of the soil and he planted a vineyard. Man previously described as a
righteous man now seeks comfort in wine. He was indeed a righteous man, one who had seen a
world destroyed. When reading the text, impression of Noach as man paralyzed with grief,
seeking oblivion. In contradistinction, read about Avraham who experienced the greatest
tragedy/ trauma= lose the most precious person in his life and just as he was about to angel of
God urged Avraham not to. Seemingly happy ending immediately overshadowed by death of
Sarah, Avraham’s wife. With prototype of Noah’s grieving process, we would assume Avraham
would grieve for a long time. Rather, Avraham mourns and then secures the Jewish future by
performing two acts- purchases Cave of Machpeila and finds a wife for Yitzchak. While Noach
is overwhelmed by his loss, Avraham grieves by knowing what he has lost but takes his
experience to strengthen the Jewish future.

Complex issues that many people suffer from silently and organized- these issues are often
ignored because human nature to avoid confrontation with____. Must break the silence. Having
been founded five years ago, Student Medical Ethics society realized there was a need in the
Jewish community for Medical Ethics + Jewish ethics. As Torah Jews, accept the privilege of
“v’yrapo y’rapeh” within the confines of halakha. Throughout the year, we develop
programming that delves into various contemporary issues- stem cell research, genetic screening,
end of life issues etc. This year’s conference would not be possible without the help of numerous
individuals- Dr. David Pelcovitz, CJF led by Rabbi Kenneth Brander, Yoni Cohen, Chezi Jessin
and Marc Spear. Enormous help of Aliza Berenholz and staff at the Events Office. Finally, Dr.
Edward Reichman’s support,advice and words of encouragement have been truly invaluable.
Thank you for always being available to speak with us etc.

Introduction to Mental Health


Rabbi Dr. Edward Reichman, Conference Chairman

Good morning, everybody. I hope you don’t mind if I just check my watch- it says 9:30 in the
morning on Sunday but there’s 400 people sitting in the room here. This is just a small reflection
since our early conference on infertility on Jewish law, I guess it was so successful that we’ve
become fruitful and multiplied. Only the fifth conference and barukh hashem extraordinarily
Wiznitzer 2

successful conference. Of course CJF and YU with their vision to appreciate the necessity for
these conferences, as much and sometimes more so than any other events on campus illustrate
the ideals of this university re: Torah u’Mada in bringing together world experts on halakha,
science and medicine. Rabbi Brander- it is his vision that has guided this conference as well- he
fortunately is at a simcha but unfortunately cannot be here today. Special honor- Fuld Family
while usually is across the continent in Eretz Israel – this year we have the tremendous zechus of
having them in person. I would like very importantly spend just a moment to thank our
copresidents, Jenny and Adiel. Even if there was no conference today, it would b ean
extrardoinarily valuable effort to see the growth these two have undergone. Medical Ethics
Society has become an incubator for leaders of the Jewish society.

Our past conferences have almost exclusively dealt with refuas ha-guf, with bodily health. It is
interesting that when you say the prayer for health of individual you say he should have refuas
hanefesh and refuas haguf- mentions mental health as well. Mentions refuas hanefesh preceding
refuas haguf. I think we’ve been neglectful in ignoring refuas hanfesh and we hope to remedy
that today. Eseential factor in bodily health, people who have better mental health are actually
healthier in body as well. In that vein, wish us all a refuas hanfesh and refuas haguf for all klal
yisrael and we should be zocheh that all of our conferences lead to refuas hanfesh and refuas
haguf for all.

Introduce our first session- Dr. Tia Powell will be moderating our first session on eating
disorders. Part of YU faculty and just now is opening the Einstein/ Cardoza Masters of Science
Bioethics program. More info pamphlets on table downstairs. Figure on national scene of
Bioethics.

Plenary I: Eating Disorders

Dr. Tia Powell

Important comments on rest of the panel- wanted to echo congratulations to our two conference
organizers- when I was in medical school years ago, I was saying how interesting issues in
bioethics were. He said great that you think that but no way for you to really put that into your
work. Things have really changed radically- to look at this room today, to see the energy and
enthusiasm of our students and look upon how these issues of ethics have been so much more
richly broken into re: what we do in terms of medicine and interweaving of medicine and mental
health. Very special thing for me eto see this large room full of people re: this. As someone who
works exclusively in bioethics and as a board-certified psychiatrist- main principles in bioethics
that we turn to again and again are AUTONOMY and Beneficence – no circumstances are these
more difficult to work with than in mental health. Diseases of the mind attack person’s ability to
act autonomously- the autonomy of the person who suffers from a direct mental illness is directly
attacked by that illness.

Any form of treatment need to benefit that patient. Eating disorders specifically have some
conflicts- issue of force-feeding for instance. Prisoners are subjected to force-feeding in political
context. This in severe cases is the treatment we are forced to offer sometimes- how to perceive
doing that in order to help foster the sense of free choice, freedom and responsibility that will
Wiznitzer 3

come with mental health- so you can see that this conflict between autonomy and beneficence is
particularly distressing to negotiate in context of mental health and eating disorders. Hand the
discussion over to our panelists.

Dr. Karen Rosewater (MD, MPH)

In Young Adult Medicine and private practice in Manhattan- I’m not the greatest with slides so
forgive me if I don’t do well.

“Ethical Issues in Eating Disorders’ – hope not too basic for people but think we need a context
to leap to discussion of ethical issues. Hopet that through this talk I will be setting that
background and leading you to understand some of the complicated factors that contribute to
this.

CASE 1- SARAH P
*15-year-old girl, 5 ft 4, 150 lbs
Always slightly overweight, teased by her classmates
Mensturates begins age 13, gained typical weight (10 lbs per year, each year over 5 years)- most
people don’t realize how much weight you are supposed to gain during pubertal process- a lot of
adults don’t know that
-High IQ, staright-a student, attends high pressure private school in NYC
-decided to go on diet age 14, saw nutritionist, learned about “healthy eating” began restricting
intake- lost 15 lbs
Got a lot of postivie feedback from family and friends because of this
-Weight loss plateau-
Decided to become vegetarian, then began eating all “non-fat” foods
Began to complain of abdominal pain with eating, saw pediatrician for evaluation, noted lost 15
more pounds in 6 weeks, was told now at “IBW” (rapid weight loss over short away of told)-
she’s told her weight is perfect for her height, Ideal Body Weigh (as used to refer to it)
Decided to cut out all dairy because MD suggested she might be lactose intolerant
Abdominal pain continued
Parents noted began skipping breakfast completely, citing “nausea” in AM
Mom called by best friend’s mother, seen t hrowing away her lunch at school
Was eating dinner at home but refusing to eat what others were eating, taking plate to room to
eat because “so much homework” – isolation eating and difficulty eating with others we often
see
Mom realized buying fewer menstrual supplies, found out last menstrual period at end of
summer
No longer going out on weekends to dinner with friends
Wearing more layers/ baggier clothes
Found out she was visiting pro-anorexia sites on the web (browsing history) - give a lot of tips
and promote solidarity on patients who are anorexic
Went back to pediatrician, now 100 lbs
Referred to gastroenterologist
Patient feels she looks fine now, not willing to add meat or dairy back into diet
Wiznitzer 4

All blood testing is normal- a lot of times blood tests will NOT reflect severity of the illness and
creates a problem where hard to determine how severe the situations is
Endoscopy is normal
Then referred to my office

DEFINITIONS_ ANOREXIA NERVOSA (AN)

- Body weight maintained at less than 85% of expected based on age and height, either
from weight loss or lack of expected gain
- Excessive fear of weight gain or becoming fat, even though underweight
- Body image disturbance, abnormal focus on body weight/ shape or lack of understanding
of seriousness of low body weight
- Amennorhea of at least 3 consecutive cycles in menarchal girls/ women (obviously not
good criteria for men)

SUBTYPES

Restricting Type- no regular engagement in binging/ purging behaviors


Binge eating/ Purging type- regular engagement in binge eating or purging (laxatives)

CASE 2- NICOLE T.

-18 year old senior, 5 ft 8, originally 140 lbs, elite creative arts public high school
- learning disability and add, high intelligence, B+ to A- average, pushes self hard, stressed
applying to college
History of separation anxiety and school phobia in elementary school
On stimulant medication for ADD, notes decreased appetite during day and increased hunger at
night
Often skips lunch, “watching” weight wants to be an actress, beginning to audition for off-
broadway
At summer arts camp, saw “anorexic” bunkmate throw up after eating
After eating pint of ice cream after school, tried purging for the first time using toothbrush,
thought would prevent gaining weight
Began buying food on way home from school and eating large amount (box of ceral, box of
chocolate chip cookies, pint of ice cream) then purging (Doesn’t need to use toothrbrush)
At routine dental exam, had 3 cavitieis after having none for many years
Parents began noting patient suddenly showering after dinner, confronted Nicole who denied
vomiting
School counselor called parents because friend reported seeing Nicole vomiting in bathroom at
school after lunch (ppl start to lose control over disorder)
Brought to pediatrician’s office, blood s drawn, potassium at 2.7

Bulimia Nervosa

-recurrent episodes of binge eating (defined by eating an amount larger than a typical person
would eat under similar circumstances within a defined time period)
Wiznitzer 5

- At least 2 bingers per week for 3 months

Lack of control around binge-eating behaviors

- Recurrent compensatory behaviors in order to prevent weight gain (vomiting, laxatives,


diuretics, fasting or severe dieting and hyperexcersizing) and excessive concern with
body image (no weight criteria for bulimia so hard to detect)

SUBTYPES

- Purging Type: self-induced vomiting or misuse of laxatives, diuretics or enemas


- Non-Purging Type: use of other inappropriate compensatory methods

EDNOS- Eating Disorder Not Otherwise Specified

Examples include:

1. In females, all criteria for AN except patient gets period


2. All criteria for AN is met except current weight is in normal range despite significant
weight loss
3. All criteria for BN are met except that the binge eating and inappropriate compensatory
mechanism occur less than 2x per week or for less than 3 months
4. The patient has normal body weight and regularly uses inappropriate compensatory
behavior after eating small amounts of food (self-induced vomiting after consuming 2
cookies)
5. The patient engages in repeatedly chewing and spitting out but not swallowing large
amounts of food
6. Binge-eating disorder

Binge-Eating Disorder (BED)

Uncontrolled binge eating without emesis or laxative abuse


Listed in the DSM-IV TR appendix as a diagnosis for further study- proposed to be included in
DSM V as separate diagnostic category
Often but not always associated with obesity symptoms

Night-eating syndrome includes morning anorexia and

Prevalence (not hard and fast numbers)

AN- 0.7% prevalence in teenage girls, 90% female


BN- 1-2% prevalence in women ages 16-35
EDNOWS- ??? likely much larger
(Estimate 4% of adolescents suffer from ED)
Incidences appear to be increasing
Wiznitzer 6

Cultural Influences

Higher prevalence in industrialized societies


-emphasis on thinness as model of female beauty
-existence of “adolescence” as a phase of life requiring separation and invidiuation (erikson’s –
identity crisis)
-what are the factors in the Jewish Community that may promote EDs?

INDIVIDUAL SPECIFIC RISK FACTORS FOR AN/ BN

Family history
Eating disorder of any type
Depression
Anxiety and anxiety disorders
Substance misuse, especially alcoholism (BN)
Obesity (BN)
Adverse parenting (especially low contact, high expectations, parental discord)
Sexual abuse
Family dieting
Critical comments about eating, shape or weight from family and others
Occupational and recreational pressure to be slim
Low self-esteem
Perfectionism (AN and to a lesser extent BN)
Early menarche (BN)

Also, menarche comes early then gained pubertal weight early, harder for younger adolescent to
cope.

Biologic Risk Factors

-Genetics – twin studies support biologic factors- especially for AN

- Concordance for AN approx 55% with monozygotic and 5% for dizygotic twins
- Concordance for BN 35% for monozygotic and 30% dizygotic
- Children of patients with AN have a 5% risk of developing AN

-Genetic association studies are evaluating polymorphisms in serotonin (5-HT) related genes in
AN

-Ghrelin receptor gene polymorphism may be related to hyperphagia (eating a lot)


Ghrelin secretion is increased in BN

-Melanocortin 4 receptor gene is hypothesized to regulate weight and appetite


-Endogenous opioids may be increase in AN leading to denial of hunger
-Families of patients with bn have higher rates of substance abuse, alcoholism, affective
disorders and obesity
Wiznitzer 7

Some relations to dopamine receptors

Common psychologic comorbidities

-bipolar disorder
-obsessive compulsive disorder

Treatment

-team approach- medical, nutritional, psychological


-different approaches: individual therapy, group therapy, family therapy
Levels of care: outpatient, day treatment, hospitalization, residential

Medical Complications in AN

-Bradycardia, hypotension, Raynaud’s syndrome


Dehydration, hypoglycemia, hyponatremia
Syncope
Loss of LV mass, reduced chamber size and contractility
Acquired mitral valve prolapsed
EKG changes- predispose you to fatal cardia arythmia
-Cerebral atrophy
Anemia, leukeopenia, neutropenia, thrombocytopenia
GI issues- delayed gastric emptying, constipation, gallstones
Osteopenia/ osteopororiss
Alopecia
Hyper-carotenemia (turning orange)
Growth arrest
Infertility, increased miscarriage, low birth rates

Medical Complications in BN

-Electroylte disturbances
Arrythymias due to hypokalemia
Esophogeal tears
Dental issues

Medically, these disorders impact so many areas. Can detect certain things via physical exam but
very difficult.

Prognosis

50% do well, 25% intermediate, 25% do poorly, better with adolescents (need early
intervention)- higher rates of mortality than other

Complicated issues contribute to Ethical Dilemmas


Wiznitzer 8

-parent has eating disorder, patient’s lack of understand re: seriousness of the risk, brain changes
that affect thinking, “infrequency” of consequences, doing “well” in other aspects of one’s life,
inability to eliminate food from one’s life, community emphasis on thinness

Summary

-eating disorders are relatively common, possibly affecting many more people than we know,
particularly since EDNOS
- create a disease state with many medical complications
Patients often misrepresent symptoms due to denial/ ambivalence about treatment, psychologic
factors (depression), shame, brain changes

Ethical Dilemmas

CASE 1- Patient Confidentiality


- How much info share with family
- How much info to share with school
- Team issues with confidentiality
- Need to avoid collusion with disorder

CASE 2- Involuntary Hospitalization


- Difference of opinions, providers, families
- Breaking of therapeutic alliance
- Are we really saving a life?

Conflict of serving patients’ wishes vs. preserving their health. Preservation of confidentiality is
issue as well.

Dr. Tia Powell: Tour de force, succinct presentation of very complicated material.

Dr .Esther Altmann

She shows a FILM- the beautification of a woman via photoshop, then placed on billboard (no
wonder our perception of beauty is distorted) via DOVE

Asking you a few questions about what you just saw- nothing diagnostic about these questions,
all your response are completely normal- check in with yourself with how you’re feeling- any of
you feeling mesmerized by that?

-Mesmerized?
-Angry?
-Sad?
-Wow, maybe they could do that to me (it’s tempting, isn’t it- it’s seductive, mesmerizing and
insidious and I think we need to keep that in mind because as I talk about issues/ subtle cultural
phenomena in Jewish community, need to keep this as backdrop- whether live in Williamsburg,
Wiznitzer 9

Monsey or Teanekc, we’re all affected by this)- how we expect ourselves to look- everybody to
keep that in mind as I talk about specific Jewish cultural phenomena.

Happy to see that there are many men in the audience- I’m going to talk about it as a woman’s
illness, clearly there are more men who are starting to manifest this disorder but going to talk
about it as woman’s issue- important to note that even men who are not suffering directly from
this illness are clearly impacted by the illness. I’m sure that all the men sitting here have a wife,
daughter, mother, aunt who may have suffered from the disorder and so you know somebody
who may have suffered from it and so it has an impact on you as well.

Anorexia nad bulimia are amongst the most emotionally and physically destructive disorders
affecting Jewish mothers- I used to say young Jewish women but now am seeing more and more
middle-aged women also suffering from it. Privilege and challenge to work with women
suffering from disorders for past 20 years. Although I treat patients from wide spectrum of
religious/ cultural backgrounds. I specialize in treating Orthodox Jews as it most closely
resonates with my own cultural and religious background. Would like to address two themes in
my work with Orthodox Jewish patients.

FIRST- some of the specific cultural phenomena I am frequently asked about


SECOND- complicated interaction between the therapist and the patients’ religious practice and
Jewish identity and how they may consciously and unconsciously influence and color the
treatment process.

Not only applicable to young women suffering from eating disorders but any therapeutic
encounter and all forms of psychopathology. Over last 10 years or so much interest in question
re: whether prevalence of eating disorder is higher in Jews in general and Orthodox Jews in
particular. Concern that more prevalent in Jewish population vs. the US population in general.
Question is why that might be so etc. I think the Jewish community across the denominational
spectrums has done a huge, very impressive job in trying to address eating disorders – multiple
efforts from multiple community centers/ organizations trying to address the problem. Some data
indicating that Jewish populations populate in-patient centers at significantly higher rates than
number of Jews in USA population.

Possible reasons-

Upper middle-class population that suffers eating disorders and can afford to seek treatment
Jews seek medical attention vs. other people in the society (at higher rates)
Data a little skewed because studies done in places with many more Jews (New York etc)
However, have been several studies that assessed Jewish women and very high rate of disordered
eating (distinguished from eating disorders)- diet at normal weight, counting calories, weighing
yourself incessantly- question is where does it go from a little bit to a lot (to a disorder)

Canadian sample of Jewish high school students were compared to non-Jewish Canadian girls. In
non-Jewish girls 18% of girls scored above a cutoff for disordered eating as opposed to 25% for
Jews. 1 in 4 girls for Jews. Comparison also made between different Jewish denominations-
Orthodox Conservative Reform and unaffiliated and no difference was found within these
Wiznitzer 10

groups. Although very worrisome Jewish problem not more specifically an Orthodox Jewish
problem. Interestingly, Jewish boys were also studied and do not report higher rates than
Canadian non-Jewish boys.

In Israel, also an issue. 47% of Israeli girls were on diet at time of study – 12% only are the ones
who are overweight.

Don’t need statistics to know eating disorders are a very worrisome problem. You’re all here
today because you know at least one person who suffered from an eating disorder. It may be a
full-blown clinical syndrome or a sub-clinical manifestation of the problem which also has a
significant impact on psychological and physical well-being. In typical Jewish tradition, I am
frequently asked why- why might there be higher rates of eating disorders and disordered eating
in Jewish community?

Several hyphotheses that reflect socio-cultural ideas in Jewish life. Come up consistently so I
will preempt questions and raise them myself-

1. Pressure of Shidduchim and the psychological impact on young women to marry and
have large families
2. Shabbat and Chagim= Overabundance of food
3. Impact of Holocaust with legacy of starvation on 2nd and 3rd generation of survivors
4. Practice of kashrut itself which separates foods into permissible and non-permissible
categories (good and bad)

Clearly important biological, intra-psychic and ___ factors. In American culture at large,
everyone does not develop an eating disorder even though all women are bombarded by media
messages of unattainable fitness ideal. Not all women in Orthodox world even though many
exposed to all four factors above. Been suggested that observant young women develop eating
disorders because have no voice/ not able to take on responsibilities of wife and mother. Some
young women worry that they are not skinny enough to start dating/ fear of being passed over for
girl with better body.

I wish these shidduch stories were merely apocryphal but sad reality is they do actually occur. I
would like to suggest that this crazy question- what size does she wear- is a red herring. Illusion
harbored by some young men and women that perfect body will produce a perfect spouse and
perfect wife. From psychological point of view, body in our culture has become a dumping
ground for emotional issues/ anxieties. Concrete way of saying that I am not sure of myself, what
I want, what I need, if I want to be in intimate relationship.

We tend not to think about shidduch from man’s point of view- young men may feel unready to
meet this developmental milestone and therefore may ask silly, superficial questions. Perhaps
men need an awarenesss of their own needs and need to express their uncertainties on women’s
sexuality and marriage in meaningful ways.

One way of thinking about the pressure of early marriage and large families is that each
community imposes expectations on young adults etc. Many students I work with on Upper East
Wiznitzer 11

Side are subjected to pressure to get into Ivy League universities (years of SAT – tutoring etc)-
not so different from pressure to go to prestigious seminary or yeshiva, rigid societal
expectations of what one ought to be doing at a particular developmental point or how one ought
or should achieve success whether good shidduch or acceptance to prestigious college no
different at all. Parents can even buffer expectations by paying attention to children’s emotional
need etc or put pressure on them.

Students need to pay attention to their own emotional needs re: when they are ready for the next
transition, whether year in Israel after high school, getting married or having a baby- not
everyone is on the same trajectory. Moving through this in unintegrated way can make one feel
more like a persona, not a person. Also discussion that within spectrum of Orthodox community,
Modern Orthodxo young women are most stressed of all because they experience academic/
career achievement and young marriage whereas Ultra-Orthodox counterparts have one of these
stressors. No doubt hard to worry about getting married and getting into medical school at the
same time.

In the best of circumstances, can create impetus to slow down and try to figure out what you
really want to put on your plate- clearly one size does not fit all.

Repeatedly suggested that Jewish rituals and festivals so entwined with lavish meals- vulnerable
to eating disorders because celebrate Shabbat/ chagim with overabundance of food. However, I
don’t think that these are the causes of eating disorders. In interests of time have to skip over
some of this.

I think excess at our Shabbat meals show: relative wealth of our time. Most of us have access to
protein, meat, fish, etc – when I think of shtetl, my own garndfather’s poverty, scrimping and
saving for special Shabbat meal because meat and fish were not daily fare. Our typical baseline
fare would be true Shabbat meals in older times so we up the ante and end up in situation of
signficiant excess.

One of my concerns is that family meals are increasingly important- I had always assumed that
shomer shabbos meals having family meals but because we often host guests/ go to our friends,
sometimes families are not having nuclear meals together- strongly suggest families have at least
one Shabbat meal together – place where you can see how your kids are eating.

I’m gonna skip a little bit and will talk about some of the ethical issues that are important to
think about in my work with observant Jews. Recently, I was on a panel with a Christian
therapist who treats eating disorders from a Christian perspective and talks about how Jesus’ love
was powerful and healing. Something compelling about this- helping someone heal from the
alienation of their eating disorder. Forced me to think more deeply about my own work with
religious patients. In contrast to the Christian therapist, I am a Jewish therapist who treats
observant Jews – I use same psychotherapeutic approach as I do with other individuals.
Longstanding worry in Jewish community that seeing a psychotherapist will make someone less
religious/ render therapy threatening to their Jewish practice.

Some good reasons for this concern:


Wiznitzer 12

-historically, psychotherapy in general and psychoanalysis in particular, has been quite hostile to
religion
- 20th century psychoanalyst view religion as regressive, infantile state so set hostile tone
When started graduate school, class in Freud, given by Jewish man from Brooklyn was
scheduled from 3-5 on Friday afternoon. I went to professor’s office to discuss dilemma. Prof
responded in gruff, unsympathetic manner and made it clear he would make no accommodations
and questioned my priorities. I remember being shaken and shocked. In my worldview, there was
no problem with doing both but form his point of view I had to make a choice.

This upsetting encounter some 25 years ago has made me think of biases, prejudices, internal
conflicts and issues therapists may sit with when they treat orthodox jews. Their ethical
obligation I feel strongly to be aware of their own feelings so as to treat orthodox jews with
openness and empathy. True for psychotherapists who ARE religious and those who are not. I
think discussion about religious experience and practice has shifted dramatically over past 10-15
years. Therapists more open to learning etc. How do hopes/ wishes of patients influence the way
they see their therapists.

Many relieved not to be working with Orthodox therapist or choose to seek therapy with
someone outside of their community so more able to explore the complex without worry re:
religious complex etc. Others who seek more frum therapist – place to think, feel and grow-
various stages of religious development. Contemporary psychoanalysis has also evolved into
more honest encounter between therapist and patient with regard to realizing complex bi-
directional processes. Example: Suppose a patient in a conflict with her mother wants to dress a
certain way- stockings off, skirts above knees, flipflops instead of shoes, bright red nailpolish-
from psychotherapy perspective normal struggle with parent in process of identity formation or
religious exploration. If comes up in middle of summer where I’m not wearing stockings, does
this have to do with me and patients’ perception of me as more nurturing, empathic figure or
does it have nothing to do with me at all? Maybe she didn’t notice I’m not wearing stockings.
These are my thoughts and whether I raise them depends on treatment with patient- question is
why she is upset, what is meaningful for her and how she is trying to fully experience and find
her own unique self- a person who is neither me or her mother.

I am ever mindful of popular expression “everyone to right of me is a fanatic and everyone to left
of me is a heretic.” Like to conclude with idea that ultimately the goal of psychotherapy is
neither to inhibit or diminish religious commitment but rather to emancipate the individual to
fully practice their Judaism in a more mindful and meaningful way. Eating disorders=
developmental crisis, enslaved by worries and fears about food and their bodies. Goal of
psychotherapy is to unlock this paralysis so can be full participants first in their own lives and
then in the lives of their families and communities.

Dr. Tia Powell- Thank you for thoughtful, evocative comments.

Rabbi Dovid Goldwasser: Distinguished doctors on the panel, distinguished members of


today’s conference, Fuld family, approximately 20 years ago sent manuscript into publisher –
group of stories together with Torah thoughts. I received a call from the publisher just a few days
Wiznitzer 13

after and he said we loved the work, beautiful, only one story that is too radical for us (we’re
gonna have to take it out). That one about anorexia and the young lady- too radical for us. I said,
no problem, send that story together with whole manuscript and I’ll be happy to take it back.
They said: what do you mean? Not willing to give up on one story. I said that story is the book
and the committee decided they’ll go out on a limb and publish that story about anorexia.
Postcript- got more feedback, correspondence on that one story than the entire book.

YU Medical Ethcis Society and fuld family have done today is one further step in bringing issues
to the forefront htat affect every single family in klal yisrael and universally. Try to condense the
material. We know it says in Avos that “Ben Bagbag said: Hafoch bo- turn over the etorah, turn
over again because it’s all there.” When we come and people of the world wish to understand
eating disorders, youth at risk, all types of adverse behaviors, the very best place is to start with
our own Torah, Talmud and responsa that gives us unbelievable insight into causes, reasons,
modes of therapy that we can all gain from.

Food is always a fight. Ask anyone in this room and they had some kind of a fight with food. Lo
al halechem lvado yichyeh adam- bread is the staple food. Bread is what one needs for the
seudah to fulfill obligation of eating meals during Sabbath and holidays and other times. Also
comes from the word “locheim” to fight and also word “milchama” - the food itself is a war.
Too much, too little- make the comment, don’t make the comment. Sit together at Shabbos table-
what about the comments? Are you going somewhere; you seem to be packing? You ate too fast,
did you finish your palte, etc. Comments that plant itself which develops and cultivates as the
person grows older and never forgot that comment.

1 or 2 examples in the Talmud where we find eating disorders. Mesachet Shabbat we say “we
cannot cause ourselves to regurgitate in the Reshus HaRabim”- can’t purge in street. Not inside
either. The reason why is because of Kavod, honor. Rashi says we’re not supposed to throw up-
reason was because people used to do it to be able to eat more. How many people standing at
smorgasbord before a wedding will say have to save a bit of room cuz main is supposed to be
something good like duck- that in itself echoes Rashi’s feelings that we are becoming so
concerned about food and consumption of food and preapation of food and new course that was
just added after you finished at the wedding- Tzeida L’Derech- French donuts in a glass case and
give you a case so give you a bag so you can go home together with your donut.

Disordered eating within our community- definitely no question that we have the same numbers
here as throughout the world. We are supposed to be insular and able to hold our own values/
ethics within our community. What happened that we begin to go according to regular society?
How did we get to point where we have to be just as concerned at YU as we are when I spoke in
Harvard? I’d like to explore a little bit:

1. Cuts across all socio-economic backgrounds, all religious backgrounds, places where cut
off from media in world (Gateshead in England), people have these learned behaviors
where affects people – number one wish of girls between 11 and 17 is to lose weight.
More than ½ of 13-year-old girls are unhappy with their image. I currently have a young
boy from one of the very religious communities who is 6 years old. At 18, nearly ¾ of
Wiznitzer 14

girls are unhappy with their image. Young girls are more afraid of being fat than nuclear
war or cancer.

What has happened? What does Torah have to say re: issues of eating disorders and body image?
We have to know how an individual is supposed to view their body and the bodies of their
children as well. Body dysmorphic disorder was first written about years ago in which the
sufferes can see themselves as completely ugly. 1-100 with 1 as ugliest where are you? Many
will say around 2 or 3; that is how unsightly I am. They say if I could only take off another 5 or
10 pounds, everything will be fine.

Researchers in UCLA took pictures and displayed them – pictures where images were blurred-
and found people who suffered from eating disorders spent time trying to see details that were
not there.

Tosfos in Meseceht Avodah Zarah eyesight connected to heart (problem on Shabbat w/ eyesight
has to be checked out because of heart issue possible)- individual may be 90 pounds and 5ft 11
may view themselves as grossly overweight. Address some of the halakha/ Jewish law and
philosophy concerning eating disorders. Utterly impossible to address even one of these
questions properly- should be an individual question of law that is asked. Bring up issue so we
can understand what is happening.

Very common question that I get- is one permitted to exercise on Shabbat? Exercise mania of
today with gyms open- wonderful for person to be in shape but a person can be so involved with
this that there can be an obsession. The only reason I got to see you is because they promised
they would drive me immediately to gym for 2 and ½ hours and they’ll wait for me till I’m done.

So Shabbat- can you weigh yourself, exercise on Shabbat, is one permitted ot cut (cutting is
related to eating disorders). Good friend Mary Cohen who wrote a book on eating disorders and
said of 500 school psychologists who were interviewed by Canadian Broadcast Company
inferred that percentage of cutters in our society is similar to those who have eating disorders. A
young man who was a cutter came to the office and looked perfect – very nice young man,
looked very well- and I said to him what would you like to talk about. He said “My body” and he
said anywhere that is currently covered up is either scarred or there’s a fresh cut. I believed him
immediately but he rolled up his sleeves to show me because he thought I was hesitant.

Are you allowed to cut yourself? HaChovel B’Atzmo. Rambam says a person is obligated to take
care of themselves- be exceedingly careful in the way that you care for yourself and your health.
Any type of self-injury naturally is a prohibition. Very interesting Lechem Mishna that maybe
only an issur d’rabanan- rabbinic transgression not Torah transgression. How that helps us- not
exactly sure- cutters say to me, does it matter if I draw blood or not. If I can make a cut without
blood would that be better?

Email I received from brilliant student in Wurzweiler:

“I’m a teen with an ED. I’ve had it together with a great depression. I was hospitalized for
depression a few times. Difficulties arose in my life. I became a cutter. It was very, very difficult
Wiznitzer 15

for me. I was what you would call a youth-at-risk par excellence. I went to a Jewish book store
and wanted to find something that would help me- something in a spiritual vein that would help
me; however, what happened was that I pickedup a book and looked at ‘self-mutilation’- led me
to one paragraph on- get this- This is mutilation – when girls get multiple earrings. That is
mutilation. Do you think a teen who reads this and cuts and burns herself will try to get any help?
She will feel like an outsider.”

Individual who suffers who wants to know what is the halakha- maybe there’s a case of the
cutting- and there is in the Tanakh, in our Bible- has to be shared. People have to know.
Extremely interesting- and a question asked to me by a therapist in Queens who goes into certain
sessions together with people who have disordered eating, particularly who have been abused-
wants to try and replicate the abusive situation (not reenactment but another idea of exactly what
had happened and how if they could face the perpetrator of the crime, how they would react)- but
therapist is very religious- is there an issue of Onaas Devarim. Is there a time where therapy
would be lost? Seeing her that Dr. David Pelcovitz and Mandell are goin to be publishing a book
‘Broken Silence’ an amazing book – this book is a must-read! Not for people who have been
touched by the problem, not by those who aren’t sure- but by everyone just to understand what is
happening.

A person would have to watch out for the guidelines- when do we press- when do we not- Dr.
Altman had mentioned shidduchim and we have a wonderful system of shidduchim. It’s
unbelievable when we need systems and YU held a certain weekend and 3 talmidim went ot that
weekend and 2 became chasanim. The question that I’ve been asked maybe within this year 100
times since Rosh Hashana- what do you have to report, what don’t you have to report. Anorexic
or builimic who has been in recovery- do we have to talk about that incident back in the 8th
grade. Someone who is on edge of eating disorder but hasn’t been termed with that clinical
diagnosis.

Zechus of speaking to HaGaon R’ Moshe Sternbuch, head of Eidah Chareidis- Mum Gadol,
Mum Beinoini, (Major Flaw, Intermediate Flaw and Minor Flaw). Would depend- certain of
those would have to be reported between first 2 to 3 outings that a couple goes on date – others
can wait. If it’s in way past and the person is far recovered, that can be considered a mum katan
that would not have to be reported. Certainly full disclosure is important- people have certain
ideas in their mind re: someone who suffered an eating disorder- do they think like we do? A
favorite question among people particularly with anorexia is do they have to eat a kzayis to fulfill
obligation on Shabbat of 3 meals- answer is yes. According to great Gaon R’ Moshe Feinstein
has to be within 3 minutes.

Yom Kippur and Tisha B’av the phone rings incessantly – as many people who email, email
because they want to know do I fast on Yom Kippur or not- in latest book I wrote ‘Starving
Souls’ I was asked by doctor in Princeton to please include chapter on spiritual conflicts. There
are spiritual conflicts- I’m supposed to be fasting on Yom Kippur but they told me because of
my eating disorder (email from South Africa), I’m going to go to Gehinnom. I will be burning in
hell. She asked me: is that true? You don’t have to fast on Yom Kippur with an eating disorder-
it’s a mitzvah for you to eat. She says back to me: “I cannot fast? I mess up everything else-
Shabbos, I mess up!- Shabbos, she told me, I hated and Yom Tov because to be truthful it was
Wiznitzer 16

the hardest time. I was in the bathroom 50 tiimes” so when I told her she doesn’t have to fast, I
messed up everything else in Judaism- I can’t even do Yom Kippur right. I spoke to R’ Elyashiv
shlita concerning this and I was asked to see a young man, 18 years old, out of the country. I
flew in especially for him- he just had returned for treatment, nothing further anyone could do-
brilliant young aspiring doctor, life interrupted and put on hold- stayed an entire day in his house
together with him speaking in his father’s study. At end of the day, father came in – as per
doctor’s orders, supposed to eat the meal. As a rule, I wouldn’t watch anyone eat so stayed in
study and dtook out book from shelf- all of a sudden mother hurries in and says he would like to
see you. While he’s eating, please stay with him.

I was sitting across from him- he had a little piece of bread- smaller than your fingernail. I saw
the battle- Locheim/ lechem/ milchama. He begins to grimace- all of a sudden, a bead of sweat
forms on his forehead. He was victorious- put the bread in his mouth, that little, little peace.
What rang in my mind was “by the sweat of your brow, you will eat bread”- and if the Torah did
not mean it then, it meants it now- 70 facets to the Torah.

Dr. Tia Powell- a few minutes left to take questions from the group

QUESTION: Question for Dr. Altman. Do you think that single-sex education plays a role in
eating disorders or coed?

ANSWER: From my observations, eating disorders in coed schools and single-sex schools- I
wouldn’t make a decision on what kind of school to go to because one more common
somewhere.

Dr. Karen: Depending on situation in any school, contagion factor may play a part.

QUESTION: Rabbi Goldwasser, there’s sometimes a closed community- someone goes out on a
shidduch and disclose something and break the shidduch as a result of it and word spreads like
wildfire. I think perhaps is it possible instead of him disclosing it it’s disclosed by a rabbi- it’s
something personal andprivate, if you don’t want the shidduch then seal your lips. In time of R’
Moshe Feinstein when divorce was considered a pigam, not nice- someone who is divorced at
that point don’t disclose till 3rd date. Losing one’s name- good name is above everything- spoke
to R’ Sternbauch- 2nd or 3rd date is serious- dating parents are different in ultra-Orthodox and
modern.

ANSWER: R’ Sternbach answered this question for Jews outside of Eretz Israel where dating
structure is different. Eating disorder should not be treated as plague we are not allowed to talk
about. There is a growing concern for lashon harah, for any evil gossip and motzi shem ra,
growing knowledge within the community and not so sure how widesperead something will be
within proper context to proper people, certainly asking shailos all along the way.

QUESTION: Dr. Karen, what do and don’t we say with regard to a person’s weight? How do we
treat that?
Wiznitzer 17

ANSWER: I think Dr. Altman and rabbi can also common on this- rather than referring to
person’s appearance, if we can concentrate on their personality- you seem great; you look happy-
something about that can be preferable.

Dr. Altman: I think we’re really not aware of how we are always commenting ona person’s
appearance when we see them. I’ve seen cases where mothers have daughters who just got out of
hospital for anorexia and everyone in shul says to them they look so great when girls are
suffering and are seriously ill. Those examples speak to how distorited our eyes are to what’s
healthy and how much social reinforcement young women get when losing weight.

Dr. Powell: I had an illness that caused me to lose weight at one time and everyone was telling
me about how I looked so great. We really see weight loss, even when it’s a significant threat to
health, as a great accomplishment.

Rabbi Goldwasser: Mishna says if a person is riding along and says “how beautiful is this tree,”
already obligated with their very own life. Chachmei HaMishna say what did they do so wrong.
R’ Feurer say that should not be the comment- how strong are its roots, how wonderful the fruits-
when we comment on external beauty, we miss the whole idea. Ilan is symbol of mishna and also
symbol of person- “Man is a tree of the field.”

QUESTION: With regards to the shidduchim pressures, what if you know of a friend of yours
who is dating someone with that disorder and that person doesn’t want to disclose it, untreated –
what are boundaries of lifnei iver and whether or not to get involved in someone else’s shidduch?

ANSWER: It would depend on what the connection is between woman and the men- is it first-
hand information, is person currently dealing with the challenge and maybe not known to 3rd
party- a lot of things that would have to be known to answer that shailah correctly. In any of
those types of things where making or breaking of marriage or engagement, consult his or her
rabbi and ask a shailah.

QUESTION: Rabbi Goldwasser, outside of open-shut case of eating on Yom Kippur do you
have cases of person eating less than a shiur but still told to make bracha achrona?

ANSWER: Number of piskei halacha- there is a kulah where a person has less than a k’zayis but
for them it is fulfilling. Person who is linebacker in Jets and person watching for shidduchim will
consume is too different menus. Halakhic eating and eating to satisfaction and those two will
sometimes interface. Intersting Ramah (R’ Moshe Isserles) if takes person longer than 3 minutes
time limit or person consumes slightly less.

QUESTION: Comment regarding shidduchim when spending hard jail time as a felon doesn’t
seem to affect propriety of shidduch as compared to bulimia, whether validity of whole system
should be reviewed- obsession of shidduchim seems to be perverted by our real realities. In these
eating disorders, reads so much about self-esteem and self-image- as its incubator the home and
the yeshiva. If it’s a major cause of eating disorder, I’m a hero because all my kids are
overweight.
Wiznitzer 18

ANSWER: You’re asking a very complicated question which would be a whole other talk.
Certainly self-esteem is a very popular buzzword and very relevant but what I was trying to
suggest is that self-esteem or feeling good about oneself comes from when parents, family and
school are attuned to where you’re at as a person and can individualize what you’re needing and
how you’re needing to grow at your pace to a larger community. When there’s a gap or
discrepancy chasm between where the child’s at and what the community expects. Going to
Israel after high school- child who doesn’t feel ready to go- doing what he thinks he should be
doing but is he feeling right about it. That’s where eating disorder will fester and reach its full
bloom. Parent who says you should spend another year at home or let’s think of something else
for you- this isn’t for everybody or your daughter could go to seminary, I’m still worried about
her eating disorder, but I’m recommending small, nurturing low-key even though you have a
very bright daughter. That’s not going to work says the parent because of shidduchim. She needs
to go to large, pressurized place. That’s’ where the problems really come in. That’s where the
self-esteem issue happens.

Dr. Tia Powell- we’re overtime- panelists, a final word?

Rabbi Goldwasser: Self-esteem issues are probably present in anbydoy I’ve ever seen in past 21
years. Omen es Hadassah- Mordechai brought Hadassah up. R’ Dessler says Omen is from the
word Emunah- and because he instilled in her a strong sense of self, she was able to rise and
become a leader of the Jewish people.

BREAK

Words of Reflection
Rabbi Dovid Fuld, Conference Sponsor

Thank you very much. The good news for me is that I’m able to be here- normally this
conference occurs at a time when my wife and I are in Eretz Israel. My youngest had a baby (our
most recent grandson) so we came back to New York and we’re here on this occasion. You can
here the tremendous degrading of my vocal cards so I’ll be very limited in how long I can harass
you from this microphone. Very legitimate, authentic, fresh air future of our people- their
associates and impact they have is a privilege to our family. Their accomplishments in restoring
communal confidence is alive, infinite in his wisdom despite constant attempts from left and
right to achieve radical mutations within that Torah. Rabbi Bila? Who has been my shadchan
with YU projects, the one whom I always have the pleasure of debating and sparring with- I
thank him from bringing us here. The comment of my wife’s involvement is a minimalistic one –
without her, I’m nothing- when I get home, she assures me that with her, I’m still nothing. I want
to make the comment of why I’ve elected and I have to tell you that I’m very didadctic and very
careful and becoming much more careful in my older years of how I distribute tzedaka. Not
everyone who collects money is comfortable for me in the role of my gabbai tzedaka. This is
more- genetics lab in Israel, 134 babies that wouldn’t have been born or would have been born
very badly, poverty, hachnasas kallah attract me- this is esoteric. Shivim panim l’torah- my
father (in law?) spent a lot of time on that Mishna with us. The Mishna speaks about a holech
b’derech and hes learning and he interrupts his learning for a moment and he says ‘what a
gorgeous tree, what a fantastic fo__?- technology with improves nature as it presents’- the Torah
Wiznitzer 19

looks at him as if he’s almost obliged to capital punishment- is god as angry as they claim
according to fundamentalist jews? My father’s suggestion was that this person was not admiring
nature, biology nor technology in sciences- the guy is traveling and learning and he stops his
mishna, his learning and stops – at end of time, not sure there are many locations on earth today
where integration of looking at that magnificent nature around us or looking at magnificent
technology (medical heatlh or mental health) finds an authentic expression other than here in
YU.

All of you remember those big bus signs- before there was Prozac, there was Mozart. Shaul
employed David to play music for him. Question nowadays does music have a legitimate place
in life of our youngsters.

Shidduchim- shows how our values have been skewed and the mutations that have taken place.
A family that has exensive problems in its Shalom Bayis (we today might speak of abusive high
schools/ alcoholics)- they might demand intervention before marriage. Dr. Altman was far too
timid in talking about authentic shidduchim issues- plastic liners on tablecloths, how much you
earn or don’t earn. Just had a conversation in these cases of mental health cases- not sure that
should invalidate a shidduch, but should only be followed after extensive counseling and after
that problem is well-addressed. The authentic voice of halakha is very clear at YU in general. My
wife and I feel very privileged to have the association with these very vibrant and magnificent
kids whose dent in Jewish life leaves us hope that there will be better days and years ahead of us.
Our association with YU leaves us with a feeling that there is still a tether/ anchor where
normative Judaism, not terribly mutated, is still available.

Putting up with me just because I sponsored this to say my thought- I thank you. To YU and
Jewish Medical Ethics group, I thank you more than you could ever understand for being able to
have this association.

Plenary II: Addictions and Substance Abuse

Mr. David Mandel

It is a privilege to be here this morning and to thank those who sponsored today’s conference and
also Jenny and Adiel, co-presidents, for organizing what is really a very special day. A lot has
been mentioned about shidduchim this morning, which is especially relevant nd I would add a
comment to what Rabbi Fuld said and what was discussed in the eating disorder session that
much more attention has to be paid that young women are at a disadvantage rather than young
men when they face tehese life challenges. Too many more questions are asked about the young
woman rather than the young man and that is simply not necessary and not right and is an issue
that needs to be addressed by paying less attention o asking so many questions.

Rav to Chiya his son: in Mesechtes Pesachim, Rav spoke to his son Chiya and implored him
‘Don’t drink drugs’ – it seems that it is well known that drugs can be taken in many forms and
this is a conversation that took place 1500 years ago. Rashi explains that Rav said to his son that
if you consume drugs it’ll be bad for your heart. 3010 years ago, Koheles (Shlomo HaMelech)
wrote “What was, will be” – “There is nothing new beneath the sun.” What was discussed, what
Wiznitzer 20

did happen, will happen again. Max Weber 100 years ago postulated that the single most
important characteristic for leaders is charisma. If one has charisma, one attains the single
greatest characteristic to be a leader. IT is debatable- this is what he postulated. Dr. Abraham
Twerski postulated (following conversation this morning re: eating disorders) that self-esteem
may be single greatest attribute and characteristic. I heard R’ Twerski say 10 years ago- how did
you write several dozen books? And his response was I really only wrote on one issue- the issue
of self esteem. I just wrote it 40 different times in 40 different ways. So if charisma is to
leadership, self esteem may be to an individual. Connection which I see between organizations
like YU which focus on leardership and individual that we are focusing on today.

Session this morning on addictions and substance abuse, participants will walk away with a well-
rounded understanding of halachic and ethical considerations that govern tehse issues of mental
health- ethics involved in treating patient when therapist knows of better-suited clinicna for the
person. That deals with issue of halakha/ morality- issue of finances- and closing three words
also struck me- how can we lower the number of teens at risk? Teens cover ages of 13-19. Many
of us working with adolescents and these issues recognize that adolescents may go into their 20s
and into their 30s or 40s- not just the adolescents we’re speaking about. Young people have
issues that go into their 20s, 30rs or 40s and we may be blaming some of the adolescents when
some of the young people beyond adolescence also have these issues.

Three distinguished panelists- Dr. Trish Attia has 30 years experience in behavioral health care.
Currently she has full-time practice in NYC, clients with co-recurring disorders and sexual
abuse- adjunct professor at YU and NYU and guest speaker at conferences in Israel, USA,
Canada and nationally published author. Substance abuse in clients and women’s addiction and
treatment through historical lens.

Dr. Trish Attia

Hi, everybody. I’m not going to be talking about shidduchim. To prepare for this, I decided I
would talk to colleagues and think about ethics as I went through my practice everyday.
Unsurprisingly the issue of ethics came up everyday almost with every patient- I want to give
you two quotes from clients about ethics in terms of substance abuse.

1. One client tried to convince me that using oxycontin was more ethical than using
marijuana because it is prescribed by a doctor.
2. He only used natural substances- poppy, things from a poppy, like all the opiates would
be considered natural, or marijuana- but he would never use oxy because ethically that’s
against his ___.

Part of concept of denial in addicts.

Professionals have ethical code- to provide a clear account of what behavior is considered
ethical, correct or right in a certain set of circumstances. Ethics provide a written description of
values, principles, standards and rules of behavior that should guide our decisions, our
procedures so that we operate ina professional and ethical way that will enhance the welfare of
our clients and everyone else that we come into contact with.
Wiznitzer 21

When I first thought about doing this speech, I decided to look at the first book that I ever bought
as I moved toward Orthodoxy myself. That book was Pirkei Avot by Rabbi Avraham Twerski.
When I looked at it, I saw that unlike being called “Ethics of the Fathers” he called his “Visions
of our Fathers” and I wanted to ask him if he could let us know. Rabbi Twerski?

Rabbi Twerski: I had to choose a title for the book- “Ethics of the Fathers’ is the name of the
volume- too cumbersome to say “Commentary on Ethics of the Fathers’ so simply for a very
practical reason we gave it another name.

Dr. Trish Attia: I realized the book was far too big of a nut to crack to use it for this
presentation. So I decided to use ethics from American Psychological Association, CASAC
(Certification for Alcohol and Substance Abuse ____) – I was going to also use the American
Psychiatric Association one but I would have to pay $60 for that one, so you’ll be without that
one today. When I looked at NASW code of ethics, there was an interesting 2008 change – it
says social workers should avoid unwarranted negative criticism of colleagues in communication
with other colleagues or professionals (lashon hara addition).

Many things that all of the codes seem to agree with- some of them are pretty standard and really
don’t have a lot of difficultly- confidentiality, sexual relationships, acknowledgement of conflict
between ethics and law. We’re not to discriminate, promote, and are to prepare for our own
demise. Have all of you done this? I know I have not. We are supposed to think if something
happens to us who is going to get the records, how will this be taken care of. Section on
derogatory language about clients- I remember years ago sitting in a treatment team meeting and
there was a mental health professional there who referred to clients as “those Axis-2s” and had
expression like sucked a lemon. I thought not only is she being so derogatory about clients but
also sending message to the younger professionals- obviously lashon hara is something we all
struggle with and I think the impact in clinical settings is really very serious.

One other very interesting thing is we’re not supposed to barter so that client who asked me to
exchange fabulous jewelry for treatment, I have to say no to- apparently it leads to exploitation
and crossing of inappropriate boundaries. Very few of us have chosen a profession based on code
of ethics. How many people here in choosing to become a psychologist or social worker thought
let me check out code of ethics before going into that profession? That’d be none? I didn’t either
but really I think we should. For example, in the NASW code of ethics there is an emphasis on
interests of client being primary (self-determination). Our interactions with potential clients is
predicated on pursuit of their interests and an understanding of their beliefs whether or not it
corresponds to others. Imperative that we are aware of our own biases when thinking of matters
of religion and our own spirituality. So that they recognize potentially a conflict between own
sense of religion/ sp irituality and where client is coming from.

Interestingly and unfortunately a client cannot be hospitalized against their will for using
massive amounts of drugs. You could say but it looks like they are on a suicidal path. But in this
state and other states where I’ve worked, concept of suicidality that would mandate
hospitalization is defined so narrowly that literally the person has to have noose around their
neck- that is law but I don’t think that is ethical.
Wiznitzer 22

-Practicing within area of interest


-Payment for services
-Physical compensation

Is this in best interest of client?

Any kind of ethical code or even the DSM-IV or soon to be V always assumes you’re going to
use your best judgment. Codes and books can’t legislate good sense. So let’s look at competence.
We’re supposed to be competent in the area with which we are going to treat the client- dr.
mandel had mentioned that I asked what if you know of a better therapist?

Let’s say you’re a new therapist/ social worker/ psychologist- of course there will be better
therapists if education and length of time doing the work means better. I think early on in
somebody’s career they should be working (my view) with interdisciplinary team and not on
their own. To ensure competence, everyone should be under supervision. Should one work with
somebody who is a drug addict/ has excessive amounts of alcohol if they don’t have training in
addiction? What if they went to a conference and heard a 1-hour presentation- is that enough?
Your ethical decision to make- something confronted with each time asked to treat a client.

I have noticed that people who do not receive supervision have 2 significant problems-

1. They don’t grow so when I referred somebody to them 20 years ago they have the same
case conceptualization as they do today- and usually the people who don’t go to
supervision also don’t go to conferences
2. On more severe end, if you don’t go to supervision, I think some of your own issues- I
know some of your own issues- get involved. Now, I am a supervision junky. I have
weekly, bi-monthly and soon I’m going to be having supervision because going for new
certification in emotionally focused therapy for couples.

Example- I started to supervise a clinician because his rabbi mandated him to get supervision.
His rabbi had been sending clients to him for a long time and his rabbi noticed that each time he
worked with somebody, the case blew up- there were crises and problems and somebody was
calling the police or suicide- even though there was a very positive outcome in the end many
times. So I started to work with him and what I found out in terms of his own background/ work
was that he was sort of an adrenaline junkie where he thrived on thrill of what would happen. He
loved on some level getting crisis call after crisis call in those 2 hours before Shabbos. He loved
that people assumed that this meant that he was really important. He wasn’t in touch with any of
those issues because he wasn’t in supervision before.

To practice without supervision I believe is unethical- and I no longer refer to people who don’t
receive supervision.

What about new techniques? If you are a competent therapist in a particular field, from an ethical
perspective it is incumbent upon you that you are up to date with latest advances in the field you
are alleging to be an expert in. I was wondering recently if I needed to learn EMDR because I
Wiznitzer 23

work with survivors of incest/ sexual abuse. About 20 years ago, we set up a trauma unit in
Holliswood Hospital and EMDR was just peaking up and it looked lik ea bunch of hocus pocus
for me. Eye Movement Desensitization and Reprocessing Therapy is really a cornerstone today
and most of the people who have worked with survivors over past 20 years are really
recommending it – I did go for training and am in process of getting certified- because I believe
that is the ethical/ responsible thing to do.

If somebody is working with children and don’t know anything about play therapy, how to look
at drawings, I think that’s unethical. And that is all in the area of COMPETENCE.

MULTIPLE OR DUAL RELATIONSHIPS- first of all, they exist all the time. If I see someone
at this conference and I also treat a family member, to a certain extent, that’s a dual relationship.
The most common one that comes up for me is that a lot of therapists etihr see me or send their
relatives to me. I doubt I’m alone in this. But that inherently is a dual relationship. The social
worker or clinician bears the ethical responsibility to always make clear which role and which
relationship they have with the client and uphold the ethical obligations of that designated role.

A common one that comes up in my supervision group (maybe that’s how I fulfill ethical
responsibility to take care of my clients when I meet my demise), include 3 young clinicians who
are also rabbis- one of the things they struggle with is what role are you in. From ethical
perspective, you have to decide and let your client know what capacity you are functioning in. I
have a client who is in a situation who is working with a rabbi who is also a certified life coach.
In his role as life coach, helping client work toward developing vocational/ social skills and
getting into school. So when he turns around and tells him to put tefillin on in morning and
shouldn’ta ssociate with non-Jews at college, the client is angry and confused. I don’t think he’s
clear on what is his role. Dual relationships are confusing to clients and can potentially take
advantage of clients.

Another thing that came up this week in my own practice- much more subtle issue- client who is
also a clinician and she said I just wanted to give you this number. That person works in XYZ
clinic and if you ever have clients with insurance you could send to her and without thinking I
said, “Oh, thank you so much” and she said feel free to use my name because I’d like to help you
if you need to get someone into that clinic. Suddenly just accepting this number- what’s my
relationship with the client? She’s now helping me and now I’m supposed to use her name- so I
handed back the number and said thank you very much. Yes, this was a client with some
boundary issues.

Looking at dual relationships/ conflict of interests is where there is harm to the patient. So if you
are attending a simcha of the client’s family, you want to be careful about it – how will they
introduce you? Assuming there isn’t a confidentiality issue because I have clients who will say
“this is my therapist- isn’t she great?”- you’re looking at harm for the patient. I had an interesting
example of this in topic I was mentioning before where a rabbi that I know well and have worked
with for 15 years sent me his daughter to work with and of course she and I talked about it a lot. I
really wasn’t so sure I should accept the referall; she often insisted. Rabbi referrals are hard to
turn down- at least for me. Treatment progressed pretty well- she is no longer addicted and she
had a pretty serious addiction she was able to move on in her life and then came the one issue
Wiznitzer 24

that she had never told anybody which was a somewhat sexual relationship which she had had
with her brother. This came up 3 years into treatment. She kept saying- I just don’t feel ready to
discuss this issue- and yet she could delineate the ways that it was getting in way of her life. I
finally said to her – do you think because of my relationship with your father I’m not feeling
comfortable? She just exploded and said: He knew about this and although he did something
about it he never talked about it and pretended it never existed. I think that’s what you would do,
too. At that point I made a referral and barukh hashem I hear she is engaged.

Obviously, we wouldn’t have sexual relationships with clients. But re: physical contact in
general- I think statistics are something like 80-85% of people who come into treatment for
addiction or mental health have some sort of abuse- in that case, we don’t know how they are
perceiving that hug/ pat on the back. I think it is somewhat dangerous to have any physical
contact with clients- straight clinical (not religious).

If you don’t collaborate with other people who are seeing your client, it’s unethical-
collaboration is an ethic and a demand.

How about no renumeration? Obviously I haven’t come across in private practice often this
issue. But years ago when I opened up a hospital in Texas I had people offering me – if you pay
me $200 per client, I’ll send you clients. More recently, I was asked to go- we’d love to fly you
down to Florida; we’re opening up a new treatment program and we heard you’re only willing to
refer clients to treatment programs that you’ve seen- I thought doesn’t feel totally ethical to me.
Maybe I’d feel more pulled to send there if I had gotten a free trip. A little harder when a
program in Israel offered the same thing but I resisted anyway.

MISREPRESENTATION- make sure your credentials are clear- if you ever see them on a flyer
with the wrong credentials, you have to correct them. And I was trying to t hink of what is the
most broadbased unethical thing that has influenced this field in the last 20-25 years and it’s
managed care. Thank you!

Mr. David Mandel – pirkei avos also says “asei lecha rav”- get yourself a mentor. Part of
training, part of learning- you can never stop learning and certainly Dr. David Pelcovitz who so
many of you know at this university has been one of my mentors and someone I’ve learned so
much from over the years. You know him as the Gwendolyn and Joseph Strauss Chair in
Psychology and Education etc- special assistant to President Richard Joel- instructor in pastoral
counseling at RIETS. We know Dr. Pelcovitz is someone who is not only an expert on so many
issues in Jewish community but also around the world on issues of trauma and many other areas
to do with children, adolescence and loss. Particularly proud of publishing a book with his
father- ‘Balanced Parenting.’ Milestone in someone’s life to publish a book together with one’s
father who is his Rav/ mentor.

Dr. David Pelcovitz

We’ll put one slide up and see if that’s enough. Thank you very much-thank you to Adiel and
Jenny for your incredible patience, amazed by how persistent and patient and wise they were.
Thank you to Fuld family for generous support.
Wiznitzer 25

Spend next 20 minutes bouncing off some of the underpinnings of ethics. Trish did a wonderful
job of some of the broad-stroke issues in terms of professional ethics and I was thinking that
rather than speak about specific issues which we will hear about from Rabbi Dr. Twerski- the
whole idea is that moral/ ethical knowledge does not translate to ethical action. We all know
people who are incredible Talmidei Chachamim – masters of whatever there is to know and yet
their menschlichkeit isn’t necessarily what makes the most sense. So I’m going to bounce off of
3 or 4 core guidelines in the literature about what is it that we know serves as prevention against
substance abuse (as parents and educators). I don’t think I’ll say anything that’s all that
surprising but I think it requires review. We should talk about what is it as parents/ grandparents
we need to know about in terms or protecting our children/ grandchildren or beyond from
substance abuse. And as educators how to partner with families on this.

RISK FACTORS

Something very wise I once heard from head of NIDA- National Institute for Drug Abuse- funds
cutting-edge research at North Shore Hospiatl once- adolescents are drawn to drugs either to feel
good or to feel better. Subset that are drawn to drugs because they are thrill-seekers as many
adolescents are. They want to get to the altered state of consciousness. Another subset are kids
who do it to feel better- self-medicating an underlying depression. Research shows that it takes
between 2-4 years and beyond about connection between underlying depressive disorder in
adolescence and risks of turning to drugs to self-medicate. But the bottom line is that very often
when we look at adolescents at risk for substance abuse, they somehow elude either their
families or communities’ bumper sticker. They somehow don’t fit the bumper sticker.

We all know that families and communities have bumper stickers. What’s your own mission
statement? I heard recently Michael Josephson, head of center for ethics, said imagine yourself
after you’re born. After 120 years you’re hearing your eulogy- imagine what you want people to
say about you and then live life backwards. Very interesting kind of point. Families have those
mission statements.

- Ivy League or Die


- Lakewood, Brisk or Mir or Else
- Chesed or Bust

What happens when you have adolescent who does’t fit this bumper sticker? Kid with ADD and
can’t make it to make incredible grades or doesn’t have focus to make it in a demanding yeshiva.
This week’s parsha is parsha with famous statement from R’ Samson Raphael Hirsch on Jewish
History- talking about Yaakov and Esav. He says world history would have been different had
Isaac sent Esav to a different school than he sent Jacob. Yaakov was somebody who was
designed for studying non-stop like Shem V’Ever yeshiva. Esav was more of an active person,
impulsive person, maybe even at risk for substance abuse (nowadays)- had he gone to the right
kind of school, had his uniqueness been fostered, had his bumper sticker been changed to fit that
kind of temperament, world history and Jewish history would have been entirely different.
Wiznitzer 26

Take-Home Message 1: Keep eyes open. Aware of risk factors and when do we see a kid whose
protective shield has eluded our children. When do we have somebody who is somehow in our
schools/ family/ somehow on the out? Find solace with marginalized peers- some will self-
medicate or feel sense of belonging with marginalized group who are also using these
substances.

Take-Home Message 2: Issue of Monitoring. The research shows very clearly that lack of
supervision triples risk. Having a relationship with our children- we know what they are doing
and we are on top of them (not smothering but must refine balance between love and limits).
When we have that, that’s when we are much more likely to be able to hold them in a place
where they are less likely to abuse substances. Study after study shows this- the key factor that
will be protective is having parents who have a close relationship yet trust. Rules without
relationships equals rebellion. If it is all limit-setting, no relationship, kids will rebel. If all love
with no limits, it’s no good.

Book that came out called ‘Narcissism Epidemic’ and shows there have been studies asking
parents- what do you want most when it omes to parenting your children. In top 3 on people’s
mattering in 1920-1980 was “raising obedient children.” From 1980 on “raising obedient
children” went down in prominence and more recently doesn’t make it on to list. Now people
value independence – in order to have a really healthy child we also need to have those kind of
limits and must find that balance.

Balance between left hand pushing away while right hand brings closer and R’ Simcha
Wasserman, brilliant educator in California, says what happens when you push child with left
hand and bring closer with right hand- you turn them around. That’s what research shows on
prevention of substance abuse- finding that balance is an art especially in this society that is not
as comfortable with left hand pushing away. Incredible medrish on pasuk in Mishlei- put limits
on your child, there’s always hope- don’t pay attention to their screaming and yelling. Midrash
says ‘the more a father puts limits on his son the more the son loves his father.’ It’s true but has
to be done with balance.

Former student of R’ Wasserman- student at Mesivta High School that he was Rosh Yeshiva of-
rule was you had to make your bed before you went to school. Now what adolescent male has
easily made his beds so of course boys never made their beds- one day he comes back from night
seder at 10:30 PM and he and his two roommates walk in because all 3 beds are made with
hospital corners. On each pillow in meticulous handwriting are five words- “Beds Made By
Simcha Wasserman.” Rabbi Wasserman never gave them a dirty look or lecture but I have news
for you- the rest of our high school career- we made our beds beautifully every morning
especially in generation that’s not so good at accepting limits and we can learn from that.

Final 2 points- one is a point of clarity and that is that the more clear we are about placing those
limits, the research tells us that adolescents want us to tell them where we stand. Not lecturing,
nagging but we should calmly be clear about how we feel about drinking/ drugs. To just say it
with clarity. There’s something for clarity. All the research says if you wake an adolescent up in
middle of night whose opinion do you value most- parents.
Wiznitzer 27

Week before last spent day in LA doing a number of meetings, lectures and community talk on
the Internet and to get ready for it I asked some colleagues in Beverly Hills- Debbie did a study
where she surveyed Modern Orthodox kids going to yeshivas- to me most interesting response
was to the following question- “When you are parents of adolescents (when you have your own
teens at home) will you have rules about the Internet, what will those rules be and how will they
differ from what your parents have?” Most of the kids said we need our parents to tell us no. No
internet after 10:30 at night- parents put some limits on us. We see things that we wish we hadn’t
seen – and waste our time in ways we wish we did not. I was shocked.

Forced a bunch of kids to come to our Museum of Tolerance- when I asked them by a show of
hands- how many of you can text with your hands in your pockets (100% raised their hands).
Then I asked them how many would welcome a dialogue that would include rules on the
internet- most of them raised their hands- I was shocked. We need clarity; clearly in research
scene as important component of prevention.

Final point and final story- here’s the two quick stories.

STORY 1- it’s about “modeling”- it’s the OU bans Kiddush clubs about 5 years ago and Rabbi
Krupka who is leader of the ban back then comes to work that Monday morning and tells me that
he gets an angry phonecall. The president of one of the larger OU shuls says rabbi, I can’t
believe it- this religion has very little joy for me and men of my shul and the one thing we really
enjoy is Kiddush club and you take that away? One of r’ twerski’s recent books (of the 60+
books he’s published) is on spirituality and connection between spirituality and substance abuse
is big. Sad to find your joy in little cup of single-malt Scotch. So R’ Krupka says risk for
adolescents- getting into drinking because they come back from rabbi’s speech/ musaf drunk-
what kind of message is that for kids? The guy hangs up on her. Then he calls up R’Krupka two
weeks later, says he’s banning Kiddush clubs. Says he took his 15-year-old son to the Garden to
see Knicks lose and there were a group of college kids really drunk so I figured teachable
moment- so I said you see what fools these kids look like when they drink like this. So my son
said- this is exactly the way you and your friends look every Shabbos morning when you come
back from Kiddush club. So I’m banning Kiddush clubs. So r’ used to say that parents aren’t
Mechanchim; they are Mashpiim- it’s what we take in without even knowing it.

Expand our bumper stickers, open to overcome societal tendency to specialize in love without
limits, we have to put limits, be careful about clarity and making incredibly clear in soft but clear
way what we expect from our children and ultimately have to be aware of invincible lessons of
Shipuah while opening ourselves up to need to reaching each child’s soul.

Mr. David Mandel- There’s a really fantastic book, less than 100 pages by R’ Wolbe- “planting
and reading a Jewish child” – must read for anyone, especially those working in this field of
mental health. Dr. Pelcovitz mentioned it and it’s been said many times that leadership is not
what you accomplish in the organization that you’re in while you’re there but rather whether
they’re doing it 5, 10, 20 years after you’ve left. Rabbit Dr. Abraham Twerski has a most
impressive bio – most impressive part of it is that people are still and will continue implemtning
so many things that he has taught us god-willing many years to come, not only from books,
Wiznitzer 28

lectures, articles but from the one or two provocative moments that we’ve heard him speak about
when other people did not.

Rabbi Dr. Abraham Twerski

Thank you so much, David. I was given a very circumscribed topic to talk about the halakhic
aspects of alcohol and drugs. Not too much to say about that. There are two volumes written
about Shechiyah and ____ b’halakha- drinking and drunkenness in halakha. And as you go
through the table of contents, there are many subjects like “at what stage can a person who has
been drinking is he permitted to daven,” “what about a shochet who drinks,” “what are criteria of
responsibility,” “what about someone who drank on the ady that he was married- does that
compromise his competency for gittin and kiddushin”- but they’re not practical. These are not
things that we are going to deal with. Things we are going to deal with are actually very narrow
and I can probably do it all in five minutes so I cannot resist the temptation to deviate a little bit
from the topic and elaborate on something Dr. Pelcovitz has said when he pointed out the
enormous importance of disciplining children by smoel docheh v’yamin mkareves.

Opportunity to do so and it’s my favorite story.

As a child, I was at age 9 a chess champion. No one could beat me. One Rosh Hashana there was
a very fine rabbi from Chicago who was our guest for Yom Tov- he asks me if he wants to play
chess. I said it’s Yom Tov. So he said so why can’t you play chess on Yom Tov. So I said okay
and I beat him both times (two games of chess).

Second night of Rosh Hashana I’m told the Rebbe wants to see you in the study. My father was
looking into a sefer and he let me stand there. You sit there and you wait until you’re recognized.
And then he looked up at me- You played chess on Rosh Hashana? He said yes, rabbi so-and-so
said you’re allowed to. He did not say a word but looked back into sefer and almost
imperceptibly tiny little shaking of head. The rabbi was right- technically it’s permissible- but
Rosh Hashana is not a day for games.

I couldn’t leave because I hadn’t been dismissed so had to wait till father said “Gei g’zunteheit.”
So he looked into his sefer and a few minutes later he looked up at me. “You’re here? Gei
g’zunteheit.” Delivering a message of discipline but not letting me walk out feeling negative
about myself.

Only a few practical issues to talk about with alcohol- we are living today in year 2010. We have
to deal with today’s reality. Just a few weeks ago we learned about Noach who Torah tells us
was a perfect tzadik. And this perfect tzadik goes out and gets drunk. Sfas Emes asks how
someone who is a perfect tzadik gets drunk- his answer is extremely important. He says Noach
knew how much he could drink without being intoxicated or negatively affected before the flood.
After he came out of the Teivah he went back to using the same amoung that he knew was safe-
what he did not realize was that it was not the same world anymore. When world changes, the
rule changes.
Wiznitzer 29

Kind of tolerance that there had been for drinking in the past cannot be dealt with today.
Particularly the halakha of drinking on Purim which seems to be a clear-cut halakha. What it
says is not to know the difference between blessing Mordechai and cursing Haman. Talmud says
when Tzadik is elevated, Rasha descends. Therfore if Mordechai is blessed, Haman is cursed- so
what is the difference? I challenge people to tell me what’s the difference when you’re stone
sober and if you can’t tell me the difference when you’re sober, what business do you have
drinking? So it’s a total misinterpretation.

But let’s say people quote the Gemara and it’s a tremendous mitzvah. Well, we don’t pasken
shailos from the Gemara or the Shulchan Aruch and virtually final posek is Chafetz Chaim and
Mishna Berurah. Chafetz Chaim quotes Ramah that one n eed not become intoxicated, drink a
little bit more than normal and then takes a nap. Now Chafetz Chaim knew Talmud as well as
anyone else and when he says that, that is the halakha for our age. And anyone who wants to be
more machmir than Chafetz Chaim, I say kol hakavod, start with lashon harah. The reason why
this is so important is that in an age where alcoholism and drug addiction is so prominent, it’s
like Noah after Tevah- it’s a different age and we have to prevent our youth from seeing that
intoxication is persmisslbe let ALONE a Simchas Torah where there is nothing anywhere that
condones excessive drinking on Simchas Torah.

Nor is it appropriate to see young people at weddings drink excessively. These are things that
should be absolutely forbidden. Same by Shalom Zachor- serve a glass of beer but not to
youngster. Anyone who gives a minor a drink (not legal aspect but halakhic) other than a little bit
of wine for Kiddush is over on Lifnei Iver- putting a stumbling block in front of this person.
Hardly a month goes by where you don’t find a case of an alcohol death due to fraternity
drinking so in an age where this goes on have to take a very close look at discipline and how do
we model for our children. And I’m glad Dr. Pelcovitz talked about problem of Kiddush clubs.
If you take a look at a Kiddush- go down to shul social hall after the people leave and see how
many kids (youngsters) go around looking for little glasses with tiny bits of whiskey left over. I
don’t think it should be allowed and people in prominent positions who drink to excess are
setting a terrible example for their children, students and followers.

In terms of treatment of alcoholism, I could talk about this for 5 hours- I believe in addition to
psychotherapy you have to have the support of people who have been there and know what to do
and what not to do. I have been involved in treatment of over 40,000 alcoholics. I will never
undertake personal treatment of an alcoholic because if he calls me at 2:30 AM because he has
an urge to take a drink, I won’t know what to tell him because I never had that compulsion. This
is why support groups of Alcoholics Anonymous and NA and OA are tremendously important-
by the way, good explanation of 12 Steps on this week’s dvar torah on TorahWeb.org – it’s
mussar in the 12 steps. But anyway, an alcoholic who is clean/ sober a week, a year, 5 years still
may have the compulsion to drink and all that is necessary is 1 drink to set him off into a
drunken spree. So Friday night he has this urge- he made Kiddush on grape juice and now house
is quiet and it is 2:30 AM and normally he would call his sponsor- so is he allowed to call his
sponsor? Clear halakhadike shaila- you must ask a posek but ONLY ONE who
UNDERSTANDS what alcoholism is about.
Wiznitzer 30

Experience that I had with one posek felt that it is pikuach nefesh and that he is permitted to call
but suggested he avail himself of phones doctors have to minimized d’oraisas to d’rabanans to
make it a lesser violation of Shabbos.

Can he go to AA meeting Friday night? If he can walk there, I don’t see much of a problem.

Without public transportation then what? Ask a shaila. One cannot say: Chas V’Shalom, it’s
Shabbos! There are cases where a posek has said if the driver is not a Jew and if someone else
opens the door so that the light doesn’t go on then under some circumstances he permitted him to
drive, not a psak halakha for everyone and everyone must ask his own posek.

Halakhos are there and we have to have our poskim to deal with them.

What’s the status of a woman who is married to an alcoholic and cannot tolerate his drinking and
asks for a divorce? Can Beis Din compel the husband, assuming that he would listen, to give a
divorce? Halakhic shaila- is he one who has never been treated and is amenable to treated. I was
at an airport in Chicago and a couple comes over to me- “I was in Gateway 8 years ago and now
we are on our way to Hawaii to celebrate our 25th anniversary and without Gateway there never
would have been a 25th anniversary”- so he got his life together. But there are other situations
with people who don’t appear motivated and wife says I can’t take it anymore.

Posek must know what likelihood is- can this man get sober, etc. Simply to make shalom bayis
can be a mistake.

The question of a person who drives drunk and is arrested for drunk driving or chas v’shalom has
an accidenet and injures somebody- what is the halakhic attitude toward that person? Should one
try to extract that person from the consequences of his drunk driving? My own position is I
regularly used to get calls from lawyers- want R’ Twerski as expert witness. “My client was
arrested for drunk driving” and I said “I will testify for the prosecution” so they stopped calling
me. I think it’s a serious mistake to take away punitive consequences of drunk driving- to bring
in lenience and chesed and rachamim is a serious mistake.

I do want to say one word about PREVENTION. Because I think we are missing one point- back
in President Reagan’s administration, Nancy Reagan launched a program, “Just Say No to
Drugs.” Well, we had a gallant time with that- now we can deal with problem of homelessness as
“Just Build A House.” So some social workers asked young people- what do you think of this
“Just Say No to Drugs” and one 14-year-old woman said: “Why? What else is there?” and that is
a powerful statement. I believe many youngsters get into alcoholism/ drugs because they don’t
have an answer to “What else is there?”

These youngsters are looking for several minutes of happiness because they don’t get it from
elsewhere. How do we provide happiness from our children, background of min Hashamayim for
our children. Right now an epidemic competing with alcoholism and drug addiction for
destructiveness and that is Internet Pornography- you have some of the finest people – the idea of
you can do away with the internet is a fantasy. You can’t do away with the Internet- filters setc.
The statement was made so beautifully by Avraham Avinu- “no yiras shamayim here”- no, I
Wiznitzer 31

know we have Glatt Kosher and Pas Yisroel and Chadosh but what kind of Yiras Shamayim is in
our home? R’ Chaim Vital successor to Arizal says person should exert greater caution with
observance of his midos than with mitzvos and aveiros. So Pas Yisroel, Chodosh, Chalav
Yisrael- all important- but we must take at least as great caution with our midos of Kaas. I
believe that any person who loses his temper in the household for whatever reason is making a
serious impression on his children which is against Torah and against Yiras Shamayim. If we
develop our life with true Midos and Mesilas Yesharim, Michtav M’Eliyahu, Orchos Tzadikim
and actively model them/ live them, not teach them, then I think we can build up the kind of
yiras shamayim in our children where they will naturally avoid the kinds of things that are
destructive. Thank you for listening.

QUESTION: Directed to Rabbi Dr. Twerski- in terms of substance abuse in adolescents, a lot of
it may be due to peer pressure which brings us back to self-esteem. Self-esteem in child in home-
can you comment?

ANSWER: Obviously peer pressure is ____ and youngsters spend perhaps more time with their
peers than they do with their parents. Want to share one anecdote- number of years ago, I was
called into an emergency meeting – a meeting at which the representatives of all of the Charedi
communities came- yeshivas and Chasidic groups and talked about the fact that we’re losing our
kids. So one person got up and he said: So what do you think, when I was in yeshiva, you think
there were no bums around then? There were bums around then but I didn’t associate witih them
because I didn’t want to lose my parents. Today’s children are in a situation where they don’t
have parents to lose. Father comes home from work at 8:30 at night, he’s got a cell phone and
beeper and youngster comes over to talk to him and says “Tatte” and he says can’t you see I’m
busy now? I’m afraid there is some truth to that. Sometimes responsibilities to care for family
financially are so great that don’t have time etc.

QUESTION: Zero tolerance to alcohol for any adolescent / at any time?

ANSWER: You don’t want to do anything that you can’t observe. Zero tolerance for alcohol I
don’t think is necessary- I think it is perfectly okay to make Kiddush on wine. For grownups who
want to have l’chaim after the Tish, that’s also legitimate- but show control. The greatest
incident of children who become alcoholics are either from parents who are alcoholic or
teetotalers because parents who never drank in a controlled fashion never showed ability to
control alcohol- we need to show responsible drinking because zero tolerance can backfire.

BREAKOUT SESSIONS

Bullying and Harassment: Medical and Psychological Effects


Rona Milch Novick, PhD (Director, Fanya Gottesfeld Heller Doctoral Program)

The slides, which were prepared less than 2 weeks ago, are already somewhat out of date.
Already more young people who have lost their lives because of bullying. Whether I’m speaking
with children or with adults primarily, I hear things like “boys will be boys” or “girls can be
cruel” or “This is just a part of childhood, isn’t it?” My response to that is: “There was once a
Wiznitzer 32

time where polio was also just a part of childhood.” Thanks to Salk and other researchers, we’ve
largely eradicated polio. I hope that we can eradicate this scourge of childhood as well.

Let me define the terms because there’s a lot of bad stuff that happens in childhood and
adolescence but bullying is the abuse of power to deliberately cause harm to another person. I
kind of cringe when I share this definition because we would like to think of childhood and
adolescence as a time of innocence where children are nice to each other. We know children are
capable of this and it does happen and let me say that sibling issues are very different than
schollyard or workplace bullying.

Three types of bullying that I want to talk about today:

Physical- causing physical harm


Emotional- hurting another’s self-esteem
Social- limiting another’s ability to join social groups or participate in social activities

The only one that is truly accepted is “physical bullying.” Teasing, name-calling and verbal
harassment- it’s often that bullying, which so many people deny or wonder if it is bullying or
not- it often results in the most horrific outcomes. People who suffer significant depression and
mental illness because of this- the taunts and torments that they described suffering was
psychological.

Least recognized form is “social bullying” – it can take the form with girls from a very young
age of saying “I made a club and you can’t be in it” – starting in pre-school or kindergarten.
Boys’ social bullying on the ball-field in sports- you can be on my team but you cannot. Social
bullying is often not recognized either by students themselves nor by educators. When you ask
mental health professionals, they don’t think to look for this type of bullying (social exclusion)
but in fact it’s exactly the same type.

“Cast of Characters”
*Bullies
*Victims
*Peer Bystanders

Caveat: All students may, at one point be bully or victim. The research studies those frequently
identified as such.

More and more research is looking not only at bullies and victims and what bullying does to
them but also students who live in the fishtank where bullying is allowed to happen- watching
people being humiliated, taunted and embarrassed. The research I will cite you on people who
were bullies and victims regard those children who are frequently bullied or victims. The
research looks at those people for whom this was a fairly significant and regular pattern.

Chronic Bullies- Debunking the Myths

 Impulsive
Wiznitzer 33

 Aggressive
 Likes controlling others
 Average self-esteem
 Average intelligence
 Minimal anxiety or depression
 Deficit area is interpersonal empathy

Bullies DO NOT SUFFER from low self-esteem. This is a critical myth to debunk because as
long as we think that bullies have low self-esteem we send them to school guidance counselors
and therapists to build their self-esteem so we just get bullies who feel better about their
bullying. Suffice it to say that bullies are impulsive and aggressive youngsters. When I say
aggressive, I want you to remember the three types of bullying and only one type of aggression is
physical. There’s a reason we call them “biting words”- bullies who are able to turn a phrase and
use sarcasm to hurt and to harm.

Social aggression looks like this- Motzei Shabbos can you invite her to come to the movies but
don’t tell that other girl that I invited you etc- bully who is bossing everyone around socially
without ever being physically aggressive.

Bullies like to be in charge and in control- average intelligence (people thought bullies bully
because they are not so smart- but the fact is that they are often average or even smart). In fact, in
one of the most recent studies of bullies’ victims and a third group, bullies looked the most
psychologically healthy of every group. Their deficit area is a LACK OF INTERPERSONAL
EMPATHY. Empathy is the skill to put yourself in another’s shoes and bullies literally do not
feel another person’s pain. The only brain imaging study of bullies looked at late adolescent and
early adult youth watching a movie of somebody playing the piano and someone else
deliberately coming along and slamming the piano lid on their fingers. Usually when I tell that
story, I can see the empathy on people’s faces.

But when they looked at the brains of twenty 17-21 year olds who had a history of bullying and
conduct problems, the part of the brain that registers pain barely activated and thte part that
registers excitement, pleasure and arousal activated. They get AROUSED by others’ pain. They
may experience a type of pleasure at that type of pain. Of course, this research was done on
brains of 17-21 year olds so it could be that being a bully for all those years trained the brain to
resonate that way or maybe this was the cause (chicken or egg). Bullies do lack empathy.

There has also been a discussion in the research literature of whether bullying is a matter of
moral disengagement or social information processing deficits. Is it that I don’t process social
information well- I can’t read your face that you’re upset (can’t tell that I’m hurting you).

“Proactive aggressive children and bullies lack not simply the cognitive ability in one or more
steps in the social Information Processing model, but any sense that victimizing others for
personal gains is morally wrong” (Arsenion & Fleiss, 1996)- maybe they don’t recognize that
this is negative.
Wiznitzer 34

People might ask- where does this come from? Are children born morally disengaged? I speak
about as often to adults as to children- invariably there are adults who wear their moral
disengagement on their sleeve while they’re talking to me and will suggest that if my child gets
ahead, what do we care. Clearly there are some elements/ culture parenting that contribute.

This just backs up the notion of who is the healthiest- over 1300 spanish middle schools and
looking at levels of self esteem, life satisfaction, depression, stress and loneliness- you see that
bullies are the healthiest in terms of how they turn out. Lower levels of depression, lower levels
of loneliness.

Unfortunately, though bullies may look very healthy, Dan Olweus talks about the cost for doing
nothing- outcome for bullies. (Norway is the first country that attacked the problem of bullying
seriously when three pre-adolescent boys committed suicide over 28 years ago because of it and
Norwegian government and Dr. Olweus says we have to change how we educated children.)

 Increased risk for substance abuse


 Increased rates of court involvement
 At age 23, significant increases in criminal behavior

2 or more arrests for violent crime by age of 23. After 3 bullying times, you were no longer
allowed in public schools and went to treatment/ detention centers in Norway.

A 2007 study called “Boy to Man” study in Finland looked at the same thing-

*2540 boys born in 1981


Info gathered at age 8 from parents, teachers and children
Psychiatric info based on military examination when subjects aged 18 to 23
Bullying-only predicted antisocial personality, substance abuse and depressive and anxiety
disorders
Victimization-only status predicted anxiety disorder
Bully-victim status predicted antisocial personality and anxiety disorder
Identified 28% of those with a psychiatric disorder 10-15 years later

What happened at age 8, you could then predict 28% correct.

“Well, how would you feel if this was done to you?” answers “I wouldn’t care”- not easy to
build empathy. So in interim while building empathy, you need to give clear, consistent
consequences. In our school/ in our family we do not do x and therefore etc.

Understanding Victims

 Reactivity is their primary issue


 Largely pre-wired, difficult to change
 Have been told to ignore, not to react
 Escalation likely following inaction
Wiznitzer 35

Victims REACT. If I laugh it off, I won’t be marked as a victim. But if you tease a person and
she cries or her lower lip quivered and you’re a bully and want power, you’ve got it. Now you
can make your victim cry. REACTIVITY is largely pre-wired. It is part of a child’s
temperament. Anyone who has tried to tell an anxious, try, reactive child “Just stay calm” is like
telling somebody to “grow”- you’re bumping up against someone’s biology.

Very many times they’re told to ignore and not react and even if they manage it for a split
second, can’t keep it up. The bully just escalates or ups the ante assuming that the victim ignores
it at first.

Chronic Victim-Characteristics

Passive

*Passive
*Shy
*Few friends
*Submissive
*Reacts to bullying with clear distress

There’s a subset of victims, however who are Provocative.

*React to bullying with combo of aggression and anxiety


*Impulsive
*Irritating to others

If the person is a provocative or bully-victim, half the time she runs crying and the other time she
fights back (throws shtender back). If adults see this, they see the victim looking exactly like a
bully. These are by far the most challenging/ puzzling children to work with- these are the ones
who have the most massive psychological issues. One minute they are in a counselor’s office
crying about a victim and the next minute it’s for aggressive behavior. Many of these children
are irritating to others (tapping etc)- overrepresented for ADD, defiant disorder, non-verbal
learning disabilities, autistic spectrum- my formal grouping for them is “Nudge-a-dik” – they are
harder to be with. It’s not on purpose/ organized. They are very challenging students and they are
the kind of victim where you say what did you want me to do; I can’t tolerate that tapping
anymore! Seems justified and acceptable.

Outcome for Victims- Olweus, 1993

*Boys victimized between 6th and 9th grades had higher levels of depression, more negative view
of self at age 23
*Degree of childhood victimization highly correlated with adult depressive tendencies
*Pattern persisted into adulthood even when no longer victimized
*If victimized between ages 8 and 14, men are more likely to have children who are victims
(Farrington, 1993)- again chicken and egg question
Wiznitzer 36

Impact on Victims- In the moment

*Psychosomatic symptoms including headache and stomach ache


*Impaired sleep
*Bed wetting
*Dysphoria
*Depression
*Suicidality

Example Rona knows- she picked first row of 6th grade- all 6 talked about depression, 2nd one
talked about suicide. 4th boy told the following story (dark-haired, dark-skinned), a year and a
half after 9/11- he lived for a year and a half in a public school being teased that his father was a
terrorist (his father was Iranian boy). That stopped and then he started to be teased about his
weight so he stopped weighting (he ate no solid food for 8 months)- he lost a lot of weight. He
got the train schedule to find out how he could cross railroad tracks to walk home how he could
lay on tracks and end it all before his mother could get him. We’re randomly picking 6th graders
at posh middle school in Long Island- we of course asked if he’s in therapy and he is, all works
out etc.

Unfortunately in America, we did not begin talking about bullying/ paying attention to bullying
until 11 years ago when Columbine happened. Bullying got linked to violence in America.
People thought about school shootings and Columbine. But for every one terrible attack like
Columbine, there are dozens upon dozens of students who are depressed who you will never hear
about.

Wolke, Woods & Bloomfileld in 2001 looked at 1639 British students from 31 schools and
found that the effects of bullying were less strong younger you are and got worse the older you
got. Children with repeated sore throat, colds, etc worries about going ot school should start
asking parents whether they are being bullied.

Beyond Hurt Feelings (Bonanno & Hymel, 2010) – Why do some victims attempt/ commit
suicide? 399 8th-10th graders exploring social hopelessness vs. social support. Social hopelessness
has the most direct contribution to suicidality vs. those who are being teased who DO have
friends (the Geek Squad, Football Team, have somebody- that decreases suicidality).

More people are looking at social mapping.

Intervening with Victims-

*Social skills training may be helpful


*Assuming a “role” may be helpful
*Decreasing or “covering” reactivity is critical – pretending allergic reaction (has to go to nurse)
*Providing safety is critical- telling reactive child (give them a job to get them out of room- run
out and get me some water etc)
Wiznitzer 37

I saw a girl years ago who gave me an amazing strategy quite by accident (she was very tall and
lanky and was playing Abraham Lincoln and had to wear a tophat- she was being bullied) and
said Dr. Novick I’m so nervous because for the play I have to stand so still and if I shake it falls
off. So you’re saying on the inside you’re feeling very nervous and shaky so on outside I have to
stand very straight. So I said could you do that when people tease you- could you play the part of
Abe Lincoln when people tease you? All the years people had been telling her: Feel different/
don’t get upset. All I did was say: Don’t ACT itself. The children are able to do this for small
periods of time.

Technological Amplification – it is bullying gone wild- amplifies everything that is bad about
bullying

*Mass and rapid dissemination


*No safe haven
*No escape
*Permanence

You can be humiliated in front of 3 million people in blink of eye/ click of finger (YouTube for
instance). Also cell phone is number one way of connecting to Internet; so monitoring what kids
are doing on the computer isn’t necessarily preventative enough. There is no safe haven- kids
who are being bullied get it literally buzzed to their hip pocket. There is also 100% permanence-
if it has ever been on the Internet, there is a trace. I cannot believe the stupidity of young people
who tell me things like one girl was harassing another girl with 100s of text messages an hour
and said but there’s no record of them- I deleted them. And I said have you ever seen your
parent’s cell phone bill? Clueless teenager- there was a question of legal proceedings against her.

I’ve had other teenagers tell me they know how to wipe it clean- and I say- “You work for FBI or
CSI?” They’re rather dumb about these things.

Bystander Issues

*85% of all children will never be victims or bullies but will watch it- they’re afraid of becoming
the victim/ discomfort with confronting bully- they do nothing

Social Psychology Phenomena

- Multiple, well documented social factors influence group and peer dynamics
- Dilution of responsibility/ Inaction phenomena
- Power to conform
- Dehumanization of victims

If we see someone dehumanized enough times we blame the victim- take it as their responsibility
rather than thinking of cruelty of others.

Attitudes that Interfere


Wiznitzer 38

*Boys will be boys


*Isn’t this just part of childhood
*He/ she should just clobber the bully- that will end this
*My child is all that matters
*I’ve told my child to mind their own business

These are adult attitudes that predominate- cover page of a lovely book called “say something”
and more and more there are webpages and books to discuss these issues in safer ways.

Our Ethical Mandate

*Include screening questions in all medical and mental health evaluations


*Respond when bystanders
*Contribute to environments where bullying is not acceptable

Rnovick1@yu.edu (if you want to contact her)

QUESTIONS: Parents who cyber-bully- parents who go online and harass either friends or
nemeses of their kids and it ends with suicide.

ANSWER: Pre-dates cyber bullying. I heard about parents carpool-bullying. “No, I won’t pick
you up” and picking up the house next door. Adults at times are markedly inappropriate. What
remains to be seen is what the legal system will do about laying the culpability at the foot of
parents who do that in a way that damages the health and mental health of children. Very
difficult to stop bad parenting. And people do that and you know, I can’t go to someone’s home
and unplug their computer and say you lost your Interent privileges, Mrs. Bully.

Shira’s email address= it could be Shira’s best friend playing a trick on you, her parents etc and
kids get devastated about it because they think it’s a kid. Need to educate kids that it is not
always the way it is.

QUESTION: What’s first line of defense for a parent?

ANSWER: If a child comes to you and say so-and-so-did-something-to-me first ask them what
you did to them first. Get the whole story. Bullying are videos, not snapshots. Then, if in fact it
turns out that your child was bullied- the number one thing to do is validate that whatever it was
that you did, it does not make how you were treated okay. The fact that you had five dollar bills
hanging out of your packets doesn’t mean it’s okay that they grabbed it from you. Beyond scope
of today’s presentation re: what parents can do if their child is bullied. Think of Hippocratic
oath- do no harm.

I had one parent who wanted to go to recess every day and play chess with her 7th day son and I
said “Mommy, not going to build 7th grade son’s popularity.” Not going to fix situation better.
Can’t always do what you want to do = I’m gonna talk to that parent, talk to that child who is
bothering my child.
Wiznitzer 39

Sixth-grade girl said this to me: Dr. Novick, I went home after these workshops and said I can’t
listen to my mother anymore. My mother always told me that if you see trouble, you walk away
from trouble. But after doing these workshops I realize if I see a friend in trouble and I walk
away, what kind of friend am I? We should be doing no less than that sixth-grader.

If we see children suffering- as teenagers, teachers, as parents- our obligation and opportunity to
raise a generation that can’t stand by.

Plenary III: Suicide & Depression

R’ Herschel Schachter

Aseres HaDibros usually assume, not necessarily correct but assume first 5 are on one of the
Luchos and the second five are on another Luchos and the Midrash draws correlation between
the ones that match up on the different Luchos. Based on assumption that corresponding dibrah
on other side to “Anochi Hashem” is “Don’t Kill”- the Chumash mentions this prohibition of
murder along with prohibition of sucide after the mabul. The chiddush is that even after the time
of the mabul when the human race had really ruined the Tzelem Elokim, you would have thought
no prohibition of murder or suicide- after the Mabul, one shouldn’t t hink these prohibitions no
longer apply so the Torah repeats again. I own my property- I have the right to my proerpty- but
I am not the owner of my own life. The Shulchan Aruch quotes that one who commits suicide is
considered as one who has violated a serious aveirah and you’re not permitted to observe aveilus
over such a person and not permitted to bury him in a regular cementary. Commentaries to SA –
accepted position in recent years- that this din/ prohibition that you don’t observe aveilus and
regular place in ceremony is only someone who commits suicide like a philosopher, like a
French existentialists who decided life is not worth living and nothing makes any sense- one who
is an apikores and that’s why you’re not allowed to bury him with anyone else.

But 99.9% of people who commit suicide are doing os out of a state of depression- they really
don’t know what they’re doing. So then you would observe regular aveilus and bury the person
in a regular cemetery. Minchas Chinuch is a popular sefer that many study- points out that the
prohibition against committing suicide is not as severe as that against one who kills someone
else. If you kill someone else, you’re chayav misah but if you kill yourself, only misah b’dei
shamayim- the bullet not only succeeds in killing this person but breaks his glasses, don’t have to
pay for damage caused. Person gets the death penalty (doesn’t have to pay for the damages that
he caused). But if he does an aveirah for which the punishment is misah b’dei shamayim or
kares, don’t have to pay. So Minchas Chinuch gives an abstract case where commits suicide and
also damages – do his inheritors have to pay for the damages he caused while committing
suicide? Minchas Chinuch says yes, they have to pay. It’s an averia but not the same level as
killing another.

Medrish that appears in Tanhuma/ Rabba that comments to a pasuk in Parshas Noach where it
says one is not permitted to take his own life- “Ach” or “Rak” – Ach and Rak are a miut- the din
doesn’t always apply; means there are exceptions to the rule. Medrish says that one is not
permitted to take his own life even as Shaul HaMelech (who committed suicide because he was
afraid he would bbe tortured to death). During the war Jews petrified Nazis would torture him-
Wiznitzer 40

the pasuk says “ach” meaning that according to the Midrash this is permissible. R’ Yosef Cairo,
who wrote Bais Yosef, commentary on the Tur, 2nd edition added additional comments called
Bedek HaBayis, so you can find it printed in bold block letters instead of being printed in mild
Rashi script- so it’s one line that the person who is afraid he is going to be tortured to death is
permitted to comitt suicide. So Bais Yosef is convinced this is the halakha but many later poskim
are not so convinced that this is the halakha.

If the person is in the army and knows secrets and is afraid that if he is caught alive then he’ll
release the secrets and will be detrimental to war effort, then that’s certainly permitted. Permitted
to kill people in war and also can commit suicide if necessary because of secrets he is keeping
that he can’t reveal. But not the same as one who is simply afraid of torture- to spare oneself
from great pain.

Famous passage in Talmud with Chananya, Mishael and Azariah refused to worship Avodah
Zarah- thrown into the fiery furnance and they survived (they didn’t die). Why did they allow
themselves to be thrown in? Because they were pressured into worshipping avodah zarah and din
is yahareig v’al yaavor. If the Goyim would have tortured them seriously, they would have
worshipped avodah zarah. (Passage in Talmud) So simple reading would be that if you torture
person long enough, not able to withsthand but Tosfos and many commentaries- Shita
Mekubetzes- saying a halakhic statement that if they had tortured Chananya, Mishael and
Azaryah would have been permissible for them to worship avodah zarah. Tosfos has one
explanation- wasn’t really Avodah Zarah. Shita Mekubetzes says to be suffering/ extreme torture
that’s more than giving up one’s life- you can just give up your life, but adiditnoal pain/
suffering/ extreme torture then you can just worship the avodah zarah. So it’s a question whether
we say like this.

We violate 610 mitzvos when one’s life is in danger (mechalel shabbos because of pikuach
nefesh) except for 3 including shefichas damim. Is it permissible to commit suicide b’makom
pikuach nefesh- if we assume extreme torture is the same as giving up one’s life – if one wants to
commit suicide to spare himself from extreme torture, it would be permissible. (I DID NOT
CATCH ALL OF THIS/ DIDN’T UNDERSTAND ALL OF THIS SO MY NOTES ARE IFFY)

Regarding depression- most of the suicide cases we’re familiar with- shoteh (deranged), katan
(before yom tov), cheirush (retarded and in days of Talmud deaf-mute wouldn’t be able to be
educated and thus would also be considered retarded). So cheirush really means a retarded
person, not a deaf-mute person.

Gemara has a discussion regarding shoteh- what is the definition of shoteh? Gemara says
someone who does something in the state of depression is considered a bar-daas and if he gets
married, needs a get, everything is fine. Hilchos Gerushin & Gittin this comment which Rashi
says on the Gemara- only if he has a light depression- but if someone has major depression who
is lying in bed all day and doesn’t function at all then certainly considered lav bar-das.
Sometimes a person is healthy and sometimes the person is in a state of depression so Talmud
says when person is in a state of shoteh considered shoteh in everything and when bar-daas then
bar-daas in everything. Big debate in 1920s or 1930s- someone who has fits of insanity
sometimes and otherwise fine- is he a shoteh or like a yashein? One who walks in his sleep and
Wiznitzer 41

does things, he’s considered a bar-daas but whatever he does, he’s not expressing any daas.
Person who gives a get while sleepwalking- it’s not valid. He’s a bar-daas but not expressing his
daas now. What if I appoint shliach to give a get and then go to sleep, that’s perfectly okay- not
considered a shoteh then – but if I become a shoteh after appointing a shliach then can’t deliver a
get while he’s in a state of shoteh. How can you label a person a shoteh if he is in and out-
sometimes he is and sometimes he isn’t? Some rabbanim felt to label a person a shoteh, only if
he is always a shoteh. Big question of halakha l’maaseh.

Another issue relevant to deep depression- in Gittin, we have someone who is a shoteh but we
have a drug that we can use to heal him. I assume there are people who are in the state of deep
depression but can take a drug, then he can be healed. One opinion is that whenever person is in
state of lav bar-daas but has the medication then he’s considered yashein (sleeping). We don’t
accept that opinion- we pasken like R’ Yochanan- the patient is still considered a shoteh. Major
dispute between Ramba(n? m?) and Tur. Ramba(n?m?) is of opinion that ___ from the Torah and
Tur is of the opinion that this would be a pasul get from the Torah and the Torah would say he is
considered a shoteh.

Sometimes people have suicidal tendencies and we have to ascertain whether these are just
superficial or if they are real. This would be considered a safek sakanah if real- which means
person would be obligated to be mechalel Shabbos to save the person’s life. The poskim say that
we don’t consider this like where hashavas aveidah doesn’t apply – where if someone throws his
property out the window because he doesn’t want it, you don’t have to give it back. This is not
an aveidah m’das, rather this is an aveidah she’lo m’das- the person is depressed and doesn’t
know what he is doing. Need to violate Shabbos or whatever has to be violated other than big 3
sins.

I ask your forgiveness, I’m very interested in hearing three other speakers but I have to run to a
wedding.

QUESTION: Bas Kol came out and praises her

ANSWER: Discussion in commentaries to Rishonim is one permitted to commit suicide if he’s


worried about avodah zarah or worrying about violating (violated) some serious aveirah. See this
by Rebbe where burial took place on Friday and he thought he was mechalel Shabbos and he
committed suicide as a kapparah- it turns out that he wasn’t mechalel Shabbos but Bas Kol says
that he was going to Olam Haba. So some commentaries say that if you commit suicide because
you know you’re guilty of an aveirah, then that’s okay so you’ll say is that halakha. (I DIDN’T
CATCH REST OF THIS ANSWER)

QUESTION: Stories of conscription into Russian army and committed suicide rather than giving
up their Judaism

ANSWER: Could be that’s equivalent of committing suicide not to worship avodah zarah but I
am not familiar at all; I’m not familiar with much history.
Wiznitzer 42

QUESTION: I intern at Bellevue and was seeing a patient in-patient who said suicidal ideation.
He said intent and plans but not the means to commit suicide and unfortunately it was getting
really close to Shabbos and I had to call for a psychiatric consult and I wondered what halakha is
with staying later with patients who have suicidal ideation

ANSWER: Pikuach nefesh, you’re obligated to be mechalel Shabbos- you have to be concerned
about a far-fetched possibility unless it is 1 in 5 million. You have to stay in the hospital and
attend the patient until it’s under control. You have to stay in the hospital – you shouldn’t go
home. There are certain circumstances where we have Jewish doctors driven home by non-
Jewish drivers…but in this case, better luck next time. (LAUGHTER)

SHABBOS AT BELLEVUE, WHAT COULD BE BETTER…

Dr. David Brent

OBJECTIVES

 What are the different types of self-destructive behavior and how are they interrelated?
 What societal, familial and individual factors lead one to commit or attempt suicide?
 Are there treatments that exist that can prevent suicide?
 What are the sequelae for those that are left behind?

TYPES OF SELF-DESTRUCTIVE BEHAVIOR

*Suicidal ideation (thinking about it, no attempt made)


*Suicide threats (verbalized, no attempt made)
*Suicide attempts
*Completed suicide
*Non-suicidal self-injury (cutting, for example)
* Health risk behaviors (kids who have unprotected sex, don’t wear seat-belts)
*Addiction

Some people talk about them as all equivalent to one another; they really aren’t. What they all
share are a lack of some of the protective factors that we heard from Dr. Pelcovitz this morning.
Tendency to act impulsively that can lead to any of these behaviors.

SUICIDAL THOUGHTS AND ACTIONS

*Suicidal ideation is correlated with depression


*As many as 15-20% of people at any one time have had suicidal thoughts
*Around 4-6% have suicidal thoughts with a specific plan and/or intent
*50-72% of those with a plan will act upon it within a year without intervention
*3-8% have made at least one suicide attempt

Most significant in elderly, when 1 in 4 attempts results in fatality.


Wiznitzer 43

Peak is during adolescence- this is when suicidal behavior is most common.

There’s a big upswing as people move into puberty and young adulthood and then rate
skyrockets in elderly (much more common in men than in women in United States).

Almost any psychiatric disorder can increase your risk of suicide

*mood disorder, especially chronic, bipolar, prominent insomnia


*panic disorder, PTSD
*substance abuse, conduct/ antisocial disorder
*schizophrenia

MEDICAL ILLNESSES THAT CONTRIBUTE TO SUICIDE RISK

Two general categories- brain diseases

*Epilepsy
*Parkinson’s disease
*Brain injury
*Chronic Pulmonary Disease
*Asthma
*Chronic Pain
*Medical treatments: interferon, retinoic acid, anticonvulsants

PSYCHOLOGICAL TRAITS

Hopelessness, impulsivity, poor social problem-solving, difficultly regulating emotion, low


distress tolerance, low threshold for emotion dysregulation

BIOLOGICAL CORRELATES THAT SEEM TO BE UNIQUE TO SUICIDE

*Low levels of serotonin in CSF


*Altered number of serotonin transporter receptors in the orbital pre-frontal cortex
*Distinct pattern from that of depression
*Serotonin levels can be altered by early childhood stress as well via genetics

FAMILY AND SOCIAL FACTORS

*Parental history of psychiatric illness and suicidal behavior


*Abuse and neglect
*Discord
*Disruption of interpersonal relationships
*Grief
*Disconnection and “drifting”
Wiznitzer 44

FAMILY AND SOCIAL PROTECTIVE FACTORS IN ADOLESCENTS

*Parent-child connection
*High parental expectations
*Parental supervision and availability
*School connection
*Moral objection to suicide
*Non-deviant peer group

Even if people feel themselves to be a burden, if they feel that it would be too painful to their
parents, that is something that is protective – we try to decrease stigma so that people will come
get treatment but some degree of stigma to committing suicide does seem to be protective. Also,
if a kid is involved with other youth involved in anti-social behavior then more likely to use
drugs/ get into trouble that will create a crisis (precipitate suicidal behavior).

In older adults it is analogous-

PROTECTIVE FACTORS IN ADULTS

*Supportive family
*Live with other people (spouse, child)
*Children at home
*Sense of connection and support
*In older people, “pride in aging”
*Sense of purpose

African-Americans have a pride in getting to a certain age and other cultural groups don’t
necessarily have that same pride and look at age as a burden, not an accomplishment.

Another factor that predisposes to suicide has to do with what’s around. If there’s a loaded gun in
the house, this increases a risk for suicide. Changing things helps- detoxification of domestic gas,
barriers for bridges, the availability of toxic pesticidies.

MOST COMMON PRECIPITANTS- what brings about suicide crisis?

*In young adolescents, parent-child conflict


*Older adolescents: peer conflict, legal difficulties
*In all adolescents: bullying
*Conern about sexual orientation
*Sexual assault/ abuse
*In older adults, relationship break-up, job loss, death of spouse, illness with pain, disability or
prominent respiratory component

MOTIVATION FOR SUICIDE ATTEMPTS

*To die
Wiznitzer 45

*To escape painful feeling or situation


*To get someone to pay attention
*To express hostility

Very important to understand what the person is trying to achieve so you can get that in a way
that is not as dangerous.

CULTURAL FACTORS AND SUICIDE

There are differences in suicides across religions.

Religious Taboo: rates low in Islam, lower among Catholics and Jews than Protestants
Social Integration: Lower rates in Indian tribes with more traditional culture intact, lower rates
among rural vs. urban Blacks
Ethnic identity and acculturation: Higher in first-generation immigrants
Social transition: example of Truk Islanders- anthropologist did a study on an island where 1 in
50 young men killed themselves. Tradition that when adolescent male left puberty had to leave
home, go to this male lodge and be trained- problem was kids started to leave the home but lodge
fell apart and so during periods of cultural change these kids were left caught in the middle- clear
example of where these cultural shifts can be resolved in fatality.

SUICIDE IMITATION

Suicide clusters do occur


Around 5% of suicides occur in time-space clusters
Suicides increase after publicity or after fictional stories, increase ___

SUICIDE IN SOCIAL NETWORKS

Rate of attempted suicide higher in social network of attempters and completers


However, individuals with psychiatric difficulties tend to befriend and marry one another
(“assertive mating”)
Close friends do not appear to imitate
However, close friends of suicide victims often develop depression and complicated grief, PTSD
(Prolonged Grief Reaction)

IMPACT OF PARENTAL SUICIDE

Increases risk for suicide around 2-4 fold


Much of increased risk antedates the actual death of the parent
Parents do not die young at random (higher rates of bipolar, substance abuse)
Youth at risk for depression, PTSD, substance abuse, health risk behaviors
Youth at higher risk for depression if had discordant relationship with parent, blame others for
death,a nd if caretaking parent was also impaired and depressed after the death

PREVENTION OF SUICIDE
Wiznitzer 46

Method restriction- Carbon monoxide in domestic gas and auto exhaust, gun control, restriction
of access to acetaminophen, bridge barriers, pesticides

Improve access and quality of care: depression recognition and treatment in primary care

Availability of less antidepressants that have fewer side effects: SSRIs vs TCA, show decline in
suicide rate across life span

Treatment of depression – by medication and psychotherapy decreases suicidal thoughts

PREVENTION OF RE-ATTEMPTS

Cognitive Behavior Therapy (CBT) that focuses on suicidal thoughts and behavior- decrease
hopelessness, improve coping and problem-solving

Dialectic Behavior Therapy (DBT)- focuses on improving distress tolerance and emotion
regulation, shown to reduce re-attempts

Some evidence that lithium is protective against suicide


Clozapine vs. olanzapine

DEALING WITH THE AFTERMATH OF SUICIDE

*Family Bereavement Program


*12 sessions with parents, youth
*6 years later, lower rate of psychopathology, complicated grief, school problems, parental
mental health issues
*Focuses on strengthening parent-child bond, positive parenting, parental mental health, child
adaptive coping, expression of grief
*Also, need to identify depression, PTSD, complicated grief

Suicide IS preventable through detection and treatment of individuals at risk and public health
measures like method restriction.

Suicide can be contagious but can be contained through coordinated effort (proper methods of
media reporting etc)

Survivors of suicide are at risk for depression, PTSD and prolonged griefe reactions, but these
can be treated and prevented by strengthening the family unit.

Dr. Harvey Kranzler

It’s a humbling experience to follow R’ Herschel Schacter and Dr. David Brent who is actually
responsible for some of the best studies in treatment of depression/ suicidal ideation that are truly
Wiznitzer 47

evidence based. I’m going to present to you a very practical scale (Dr. Brent is part of the
development of this scale).

COLUMBIA-SUICIDE SEVERITY RATING SCALE (C-SSRS) – in a very short time it gives


you a sense of what is real risk and what is less risk. You always needs to take seriously if a
friend, colleague or congregant in gmail chat or instant messenging or letters or emails – this
methodology is evidence- based. Going to be used in fire departments, police departments, very
practical.

Columbia-Suicide Severity Rating Scale (C-SSRS)

 Developed by leading experts (with Beck’s group)


 Evidence based
 Feasible, low-burden- short administration time
 Assesses both behavior and ideation – uniquely addressing the need for a summary
measure of suicidality
 Comprehensive measure ethat includes only the most necessary suicidality characteristics
(most essential evidence)

SIMPLY

1-5 rating for suicidal ideation (from a wish to die to an active thought of killing oneself with
plan and internet)
-Can be two questions;

*Have you wished you were dead or wished you could go to sleep and not wake up?
* Have you actually had any thoughts of killing yourself?

If answer is “no” to both, no more questions on ideation and you move on to behavior questions-
there are four behaviors that are assessed, few questions required (figure out how to classify the
behaviors.) Somewhat cookbook but also semi-structured.

IMPROVED ASCERTAINMENT- DEFINTIONS ARE IMPORTANT

All items include definitions for each term and standardized questions for each category are
included to guide the interviewer for facilitating improved identification- if you want to use this
scale, you DO need some training. Behavioral definitions come from Columbia Suicide History
Form and Burke Ideation Project

*Semi-structured interview/ flexible format


*Questions are provided as helpful tools- not required to ask any or all questions- just enough to
get appropriate answer
*most important – gather enough clinical information to determine whether to call something
gsuicidal or not
*If it is established that a patient has not engaged in any suicidal behavior and/or ideation, then
no further questioning is required
Wiznitzer 48

Key is helping a youngster, child, adolescent, adult be safe when in midst of suicidal state.

Example…

Clinician: Have you made a suicide attempt?


Subject: Yes, I took 50 pills because I actually wanted to die.

In that case, you have enough information.

INFORMANTS/ SOURCES OF INFORMATION

-Any source of information that informs your clinical judgment and gets you the most clinically
meaningful response
-Typically subject can provide best info but if person won’t talk to you need to find out best
information

SUICIDAL IDEATION

1. Wish to die – Have you wished you were dead or wished you could go to sleep and not
wake up?
2. Active Thoughts of Killing Oneself- Have you actually had any thoughts of killing
yourself?

If NO to both these questions, Suicidal Ideations Section is finished. If YES to active thoughts,
then ask the following three questions.

3. Associated Thoughts of Methods- Have you been thinking about how you might do this?
4. Some Intent- Have you had these thoughts and had some intention of acting on them?
5. Plan and Intent: Have you started to work out or worked out the details of how to kill
yourselves? Do you int4end to carry out this plan?

Auditory hallucination that tells you to kill yourself- that’s intent.

“INTENSITY” OF IDEATION

Once determine types of ideation, few follow-up questions about most severe thought

-Frequency
-Duration
-Controllability
-Deterrents etc

If you hear that someone says “I can’t tolerate the pain” that’s a major red flag. That goes with
psychic pain and psychiatric conditions.
Wiznitzer 49

CLINICAL MONITORING GUIDANCE

For intensity of ideation, risk is greater when thoughts are MORE frequent, longer duration,
stopping the pain

There is a next step regarding questions that you need to ask further. You go to suicide attempt
definition.

“A self-injurious act committed with at least some intent to die, as a result of the act”

There does not have to be any injury or harm, just the potential for injury or harm (e.g., gun
failing to fire)
Any “non-zero” intent to die- does not have to be 100%
Intent and behavior MUST be linked
Intent can sometimes be ineferred clinically from the behavior or circumstances

For example, if someone denies intent to die, but they thought that what they did could be lethal,
intent can be inferred.

“Clinically impressive” circumstances, highly lethal acts where no other attempt can be inferred

A suicide attempt beings with the first pill ingested or cut

ABORTED ATTEMPT

When a person begins to take steps toward making a suicide attempt, but stops themselves before
they actually have engaged in any self-destructive behavior (indicator of seriousness even if
nothing happened)

PREPARATORY ACTS OR BEHAVIOR

-Definition – any other behavior (beyond saying something) with suicidal intent
-Examples- “collecting or buying pills”/ buying a gun

LETHALITY- ONLY FOR ATTEMPTS

To assess the lethality of the attempts described in the previous section.

This section assesse both:

-Lethality of the actual attempts what actually happened in terms of medical damage
-Where what could have happened

WHY POTENTIAL LETHALITY


Wiznitzer 50

Likely lethality of attempt if no medical damage. Examples of why this is important are cases in
which there was no actual medical damage but the potential for very serious lethality- laying on
tracks with an oncoming train but pulled away before train ran you over- quite lethal.

0-2 if behavior is likely to result in injury or not etc.

REMINDER- IDEATION AND BEHAVIOR MUST BE QUERIED SEPARATELY

You ask both about thoughts and actions. Just because thought is denied doesn’t mean that there
isn’t a suicidal ideation. You need to ask about it regardless of a lack of ideation.

For people who work on hotlines, for rabbis who counsel many people, for students whose
friends sometimes talk about these things and for clinicians, I’ve started using this scale in my
private practice- and this literally just takes 2-5 minutes depending on information that you get.

TIME FRAMES: LIFETIME

Behavior is “ever”- capture all lifetime occurrences


For ideation and intentisy of ideation- you want to know when they were feeling the most
suicidal etc

ISSUES AND CONCERNS

A lot of people are afraid to talk about suicide because they think if you ask the questions, you
plant the thought- all evidence points away from that.

Feasibility- you want to find out degree to which it is feasible for them to do this
Inclusion/ exclusion varies
Who can administer the C-SSRS? Anyone who has gone through this training and has some
patient contact experience; you do not have to be a mental health professional- it’s being used all
over the world

This is highly evidence based – this is a suicide assessment version which goes through quick
version.

RISK FACTORS- previous suicide attempt & psychiatric disorders (90% of people who die of
suicide is because they have psychiatric disorder)- hopelessness is an important point- if
someone is hopeless, you want to try to give them hope but also want to get them to some help

Social stressors are risk factors.

Dr. Victor Schwartz


Presenting on “Suicide and Depression in the College Population”

Why is this relevant?


-This is a college
Wiznitzer 51

-Value of studying this for general population – this is a self-contained group


-Might be easier to explore risk
-Might be easier to study interventions

DEPRESSION BASICS

15-20% report current or past history of depression (ACHA, Healthy Minds study)
40-45% report being so distressed at some time in past year that was difficult to function
(ACHA)
36% of students with positive screen for depression sought treatment (Healthy Minds)
23% meet criteria for substance abuse or dependence (CASA)

Only about a third of these students are either in treatment or going to treatment. 1 out of 4 or 5
meet criteria for substance abuse.

SUICIDE BASICS

About 10% of college students report suicidal ideation in past year (ACHA, Healthy Minds,
Penn State)
1-2% report suicide attempt in prior year
7.5/100,000 die by suicide
Suicide is 2nd leading cause of death among college students (after accidents)

SUICIDE STATISTICS

10-14 years- 1/100,000


15-19 years-7/100,000
20-24 years/ 12.7/100,000 (NIMH, 2007)
College students-7.5/100,000 (Big 10 study)
Suicide is less common in students- attending school is actually protective

Mort Silverman, originator of the Big 10 study, said that being in college is actually protective
against suicide (if you are managing well enough).

POINTS FOR REVIEW

Suicide rates increase significantly thru adolescence


Appelbaum rule for every suicide there are 100 attempts and 1000 thinking about suicide (this
has important policy ramifications)
Suicide is 2nd leading cause of death in college students, 3rd leading cause of death in non-college
But, suicide rate is lower in college students! (this is because overall rate of mortality is lower)

So shouldn’t just send students home from school because they are suicidal. Challenge for those
of us who work in the clinical arena is to understand who is at risk and what interventions we
need to call to address this.
Wiznitzer 52

There was just a paper published in “Suicide and Life-Threatening Behavior”- program initiated
by the IDF – they became concerned by rate of suicides in active-duty suicides – and a cleverly
simple maneuver among other things was stop letting soldiers take their guns home when they
went home for Shabbos. During 2 years of this program, suicide rate dropped by 40%. So means-
restriction is a very important element of suicide prevention programs.

Why do we see the rates increasing as much as we do through adolescence and adulthood? Axis I
illenssess tend to manifest. Increases in substance use. Maturation – growing up is hard dto do: a)
increasing separation from family and home b) relationships c) sleep/ appetite self-management

There’s ongoing legislation about making guns more available on college campuses- Dr. Brent is
shaking his head which is the same response all of us who work in this area all have. Just had a
piece in Huffington Post about this- the concern is that when you have 40% of your college
population having alcohol binges, not comforting to think about some of that population walking
around with guns. Social-skills training has ramifications in elementary school and high-school
populations.

In a college community, one of the things we talk about a lot is the need for Gatekeeper
Training- fundamentally important that people in generic colleges (athletic coaches, dorm
counselors, chaplaincy- people who have day-to-day contact with students need to be trained in
identification and referral of students at risk).

There’s some good community-wide programs re: management of substance abuse in the
community. Multipronged approach where have people from police to schools involved in
checking IDs, doing stops for drunken driving, these community efforts are helpful and
preventative to alcohol use.

PREVENTION-CONCLUSIONS

You don’t prevent suicide by focusing directly on suicide but on related factors
Good medical/ psychiatric care
Community focus on health and mental health
Identifying and supporting those “at risk” (both educationally and psychologically)

When you have good academic support, good social connections etc.

Is there a Jewish connection here?

-Safety/ prevention/ means restriction (meakeh)- need to make our buildings safe, our
communities safe (pre-cursor to means restriction possibly)

-Support for the at-risk member of community (ger, yatom, almana)- running theme about
protecting, judging fairly the people whom we would consider to be at-risk or outsiders in the
community such as resident alien, orphan or widow

Need to be protective and caring of people who are most at risk.


Wiznitzer 53

For further reading:

General info re suicide-

NIMH Website
American Foundation for Suicide Prevention
‘Why People Die by Suicide’ by T. Joiner
‘Night Falls Fast’ by K. Jamison
The Jed Foundation (jedfoundation.org)
‘Mental Health Care in the College Community’ by myself and J. Kay (shameless self-
promotion)

Next conference is scheduled for February 13th on Population Genetics.

You might also like